Длина стороны квадрата 6 см его площадь равна площади прямоугольника: Длина стороны квадрата равно 6 см. Его площадь равно площади прямоугольника. длина которого 9 см….

Содержание

Площадь квадрата. Площадь прямоугольника 8 класс онлайн-подготовка на Ростелеком Лицей

Эталон площади

 

Напомним, эталоном длины является отрезок длиной в 1 мм, 1 см, 1 км и т. д.

 

А что такое эталон площади? Это квадрат, сторона которого равна: 1 мм, 1 см, 1 м и т. д. Такой эталон длины называется квадратным миллиметром, квадратным сантиметром, квадратным метром, квадратным километром.

Обозначение: ,  и т. д.

Площадь  геометрической фигуры – это положительное число, которое показывает, во сколько раз эталон площади уместился в данной фигуре. Таким образом площадь  – это результат сравнения с эталоном площади.

 

Свойства площади

 

 

Предположим, что мы имеем квадрат со стороной . Чему равна площадь такой геометрической фигуры? (См. Рис. 1.)

 

Рис. 1. Квадрат со стороной

Площадь такой геометрической фигуры равняется квадрату ее стороны: .

Такое свойство площади мы принимаем без доказательств. Однако поясним его.

Пусть выбран эталон длины 1 мм. Это означает, что на стороне квадрата укладывается  штук таких эталонов длины, при этом число  может быть любым положительным числом.

Свойство утверждает, что в квадрате со стороной  уложится  штук эталонов длины. В нашем случае эталон длины – . По-иному, площадь квадрата равна . Интересно заметить, что если  – иррациональное число (например ), то площадь  – натуральное число.

Итак, мы знаем свойство площади, что площадь квадрата со стороной  равна .

Рассмотрим другие свойства площади.

Равные многоугольники имеют равные площади.

Предположим, треугольник  равен треугольнику , тогда площадь первого треугольника равняется площади второго треугольника (см. Рис. 2).

Рис. 2. Равные треугольники

Следующее свойство площади.

Пусть многоугольник разрезан линиями, т. е. составлен из других многоугольников, таким образом, что общими у этих многоугольников являются только точки сторон, тогда площадь составного многоугольника равна сумме площадей составляющих его многоугольников (см. Рис. 3).

Рис. 3. Разрезанный треугольник

Итак, мы повторили три важных свойства площади. Они используются при выводе формул для площади.

 

Площадь прямоугольника

 

 

Теорема о площади прямоугольника:

 

Площадь прямоугольника равна произведению его смежных сторон.

Доказательство: (см. Рис. 4).

Рис. 4. Иллюстрация к доказательству теоремы о площади прямоугольника

Рассмотрим квадрат со стороной .

Его площадь с одной стороны равна .

Разрежем этот квадрат двумя линиями и получим два квадрата (со стороной  и  соответственно), а также два прямоугольника со сторонами , .

  • Площадь квадрата со стороной  – .
  • Площадь квадрата со стороной  – .
  • Площадь прямоугольника со сторонами ,  – искомая площадь .

Таким образом, имеем:

Получили уравнение для . Решим его.

Что и требовалось доказать.

Итак, если имеем прямоугольник со сторонами  и , то его площадь равна .

В доказанной теореме используются три величины: ; ; . Если мы зададим две из них, то получим третью.

 

Задачи

 

 

a) Найти , если ; .

 

Решение

Ответ: .

b) Найти сторону , если площадь прямоугольника равна , а сторона  равна 4 м.

Решение

Ответ: 8 м.

c) Найти сторону , если площадь равна , и .

Решение:

, так как это квадрат.

Ответ: .

d) Квадрат с неизвестной стороной равновелик прямоугольнику со сторонами 18 и 8. Найти сторону квадрата.

Решение

Равновеликие фигуры – фигуры с равными площадями.

Ответ: 12 см.

e) Теплица имеет форму прямоугольника, одну сторону увеличили в полтора раза, вторую – в два раза. Во сколько раз увеличилась площадь теплицы?

Решение

Пусть  и  – стороны исходного прямоугольника. После увеличения стороны стали  и  соответственно.

 – площадь исходного прямоугольника.

 – площадь полученного прямоугольника.

Ответ: в 3 раза.

f) В прямоугольнике сторону , равную пяти метрам, увеличили на 20 %, сторону , равную десяти метрам, уменьшили на 20 %. Найдите длины сторон получившегося прямоугольника и сравните их площади.

Решение

Сперва найдем стороны нового прямоугольника. Сказано, что длина первой стороны увеличилась на 20 %.

5 м – 100 %

 м – 120 %

Таким же образом найдем вторую сторону:

10 м – 100 %

 м – 80 %

Итак, стороны известны, найдем площади.

Значит, исходная площадь уменьшилась на .

Ответ: 6 м; 8 м; площадь уменьшилась на .

g) Решим аналогичную задачу в общем виде.

Одну сторону прямоугольника увеличили на 20 %, а вторую уменьшили на 20 %, изменилась ли площадь прямоугольника? Если да, то на сколько?

Решение

Обозначим длины сторон исходного прямоугольника  и .

1. 

 и  – длины сторон получившегося прямоугольника.

2. 

Значит, площадь нового прямоугольника уменьшается на 4 %.

Ответ: а) изменилась; б) уменьшилась на 4 %.

Интересно заметить, что обе стороны изменились на 20 %, но площадь уменьшилась на 4 %.

 

Заключение

 

 

Итак, мы рассмотрели площадь прямоугольника и площадь квадрата, вспомнили свойства площадей. По одному из свойств площадь квадрата со стороной  равна . Пользуясь свойствами площадей, мы доказали теорему: площадь прямоугольника со сторонами  и  равна . Мы доказали эту теорему и решили типовые задачи на нее.

 


Доказательство формулы площади квадрата

Дано: квадрат со стороной .

Доказать: .

Доказательство

Число  может быть любым.

Первый случай

Пусть , где . Возьмем квадрат со стороной 1 – это эталон. Разобьем его на  равных квадратов и по свойству площадей имеем:  – это площадь эталона, с другой стороны, она равна , где  – площадь искомого квадрата со стороной  (см. Рис. 5). Отсюда получаем искомую площадь :.

Рис. 5. Иллюстрация к доказательству теоремы о площади квадрата (первый случай)

Что и требовалось доказать.

Примечание

В эталоне каждая из смежных сторон имеет длину 1. Она разбита на  частей, и тогда квадрат  разбит на  одинаковых частей, т. е. квадратов с искомой площадью . Искомую площадь  квадрата со стороной  сравнили с эталоном, площадью квадрата со стороной 1, например 1 м, т. е. квадратным метром, и получили, что искомая площадь  равна . Что и требовалось доказать в первом случае.

Второй случай (см. Рис. 6)

Рис. 6. Иллюстрация к доказательству теоремы о площади квадрата (второй случай)

Пусть  – конечная десятичная дробь с  знаками после запятой.

Тогда , , то есть это натуральное число. Каждую из сторон квадрата со стороной  разобьем на  равных частей:

Квадрат с искомой площадью , разобьется на  равных квадратов со стороной  и площадью . Тогда искомая площадь  равна:

Что и требовалось доказать.

Пояснение на конкретных числах: (см. Рис. 7)

Рис. 7. Иллюстрация к пояснению

Пусть ;

Сторону, равную 2,14 искомого квадрата, разделили на 214 равных частей.

Отношение стороны к :

Это сторона малого квадрата, а таких квадратов  штук.

Тогда , а площадь  равна .

Подставляем: .

Что и требовалось доказать.

Третий случай

Пусть  – бесконечная десятичная дробь. К такому числу  можно приближаться меньшими рациональными числами. Например: ; ; ;…; , т. е. мы отбрасываем все знаки, начиная с .

Пусть , тогда число  заключено в пределах: .

Разъясняющий пример:

 

 

Имеем:

Значит, для искомой площади  квадрата со стороной  имеем: .

Искомый квадрат вмещает в себя квадрат со стороной  и является частью квадрата со стороной , (см. Рис. 8),  пока фиксированно.

Рис. 8. Иллюстрация к доказательству теоремы о площади квадрата (третий случай)

Пусть теперь  стремится к плюс бесконечности (), тогда , .

То есть , что и требовалось доказать.

Итак, доказано, что площадь квадрата со стороной , где  – любое положительное число, равна .

 

Список литературы

1. Геометрия, 7–9 классы, Атанасян Л.С., Бутузов В.Ф., Кадомцев С.Б., Позняк Э.Г., Юдина И.И. – 15-е изд. – М.: Просвещение, 2005.

2. Александров А.Д. и др. Геометрия, 8 класс. – М.: Просвещение, 2006.

3. Бутузов В.Ф., Кадомцев С.Б., Прасолов В.В. Геометрия, 8 класс. – М.: Просвещение, 2011.

4. Мерзляк А.Г., Полонский В.Б., Якир С.М. Геометрия, 8 класс. – М.: ВЕНТАНА-ГРАФ, 2009.

 

Дополнительные рекомендованные ссылки на ресурсы сети Интернет

1. Интернет-сайт math-prosto.ru (Источник)

2. Интернет-сайт «Курсотека» (Источник)

3. Интернет-сайт «Школьный помощник» (Источник)

 

Домашнее задание

1. Дан прямоугольник, у которого  и  – смежные стороны. Найдите площадь прямоугольника, если:

1. ; ; 2. ; ; 3. ; ;

4.  ; ; 5. ; ; 6. ; ;

7. ; .

2. В прямоугольнике одна сторона на 2 см больше другой. Найдите стороны прямоугольника, если его площадь равна .

 

Прямоугольную форму площадь прямоугольника. Как узнать площадь прямоугольника

Урок и презентация на тему: «Периметр и площадь прямоугольника»

Дополнительные материалы
Уважаемые пользователи, не забывайте оставлять свои комментарии, отзывы, пожелания. Все материалы проверены антивирусной программой.

Обучающие пособия и тренажеры в интернет-магазине «Интеграл» для 3 класса
Тренажер для 3 класса «Правила и упражнения по математике»
Электронное учебное пособие для 3 класса «Математика за 10 минут»

Что такое прямоугольник и квадрат

Прямоугольник – это четырёхугольник, у которого все углы прямые. Значит, противоположные стороны равны друг другу.

Квадрат – это прямоугольник, у которого равны и стороны, и углы. Его называют правильным четырёхугольником.

Четырёхугольники, в том числе прямоугольники и квадраты, обозначаются 4 буквами – вершинами. Для обозначения вершин используют латинские буквы:

A, B, C, D

Пример.

Читается так: четырёхугольник ABCD; квадрат EFGH.

Что такое периметр прямоугольника? Формула расчета периметра

Периметр прямоугольника – это сумма длин всех сторон прямоугольника или сумма длины и ширины, умноженная на 2.

Периметр обозначается латинской буквой P . Так как периметр — это длина всех сторон прямоугольника, то он периметр записывается в единицах длины: мм, см, м, дм, км.

Например, периметр прямоугольника АВСD обозначается как P ABCD , где А, В, С, D — это вершины прямоугольника.

Запишем формулу периметра четырехугольника ABCD:

P ABCD = AB + BC + CD + AD = 2 * AB + 2 * BC = 2 * (AB + BC)

Пример.
Задан прямоугольник ABCD со сторонами: AB=СD=5 см и AD=BC=3 см.
Определим P ABCD .

Решение:
1. Нарисуем прямоугольник ABCD с исходными данными.
2. Напишем формулу для расчета периметра данного прямоугольника:

P ABCD = 2 * (AB + BС)

P ABCD = 2 * (5 см + 3 см) = 2 * 8 см = 16 см

Ответ: P ABCD = 16 см.

Формула расчета периметра квадрата

У нас есть формула для определения периметра прямоугольника.

P ABCD = 2 * (AB + BC)

Применим её для определения периметра квадрата. Учитывая, что все стороны квадрата равны, получаем:

P ABCD = 4 * AB

Пример.
Задан квадрат ABCD со стороной, равной 6 см. Определим периметр квадрата.

Решение.
1. Нарисуем квадрат ABCD с исходными данными.

2. Вспомним формулу расчета периметра квадрата:

P ABCD = 4 * AB

3. Подставим в формулу наши данные:

P ABCD = 4 * 6 см = 24 см

Ответ: P ABCD = 24 см.

Задачи на нахождение периметра прямоугольника

1. Измерь ширину и длину прямоугольников. Определи их периметр.

2. Нарисуй прямоугольник ABCD со сторонами 4 см и 6 см. Определи периметр прямоугольника.

3. Нарисуй квадрат СEOM со стороной 5 см. Определи периметр квадрата.

Где используется расчет периметра прямоугольника?

1. Задан участок земли, его нужно обнести забором. Какой длины будет забор?


В данной задаче необходимо точно рассчитать периметр участка, чтобы не купить лишний материал для постройки забора.

2. Родители решили сделать ремонт в детской комнате. Необходимо знать периметр комнаты и её площадь, чтобы правильно рассчитать количество обоев.
Определи длину и ширину комнаты, в которой ты живешь. Определи периметр своей комнаты.

Что такое площадь прямоугольника?

Площадь – это числовая характеристика фигуры. Площадь измеряется квадратными единицами длины: см 2 , м 2 , дм 2 и др. (сантиметр в квадрате, метр в квадрате, дециметр в квадрате и т.д.)

В вычислениях обозначается латинской буквой S .

Для определения площади прямоугольника необходимо длину прямоугольника умножить на его ширину.
Площадь прямоугольника вычисляется умножением длины АК на ширину КМ. Запишем это в виде формулы.

S AKMO = AK * KM

Пример.
Чему равна площадь прямоугольника AKMO, если его стороны равны 7 см и 2 см?

S AKMO = AK * KM = 7 см * 2 см = 14 см 2 .

Ответ: 14 см 2 .

Формула вычисления площади квадрата

Площадь квадрата можно определить, умножив сторону саму на себя.

Пример.
В данном примере площадь квадрата вычисляется умножением стороны АB на ширину BC, но так как они равны, получается умножение стороны AB на AB.

S AВСО = AB * BC = AB * AB

Пример.
Определи площадь квадрата AKMO со стороной 8 см.

S AKMО = AK * KM = 8 см * 8 см = 64 см 2

Ответ: 64 см 2 .

Задачи на нахождение площади прямоугольника и квадрата

1.Задан прямоугольник со сторонами 20 мм и 60 мм. Вычисли его площадь. Запиши ответ в квадратных сантиметрах.

2. Был куплен дачный участок размером 20 м на 30 м. Определи площадь дачного участка, ответ запиши в квадратных сантиметрах.

Площадь прямоугольника, как не будет дерзко звучать, но это важное понятие. В повседневной жизни мы постоянно сталкиваемся с ним. Узнать размер полей, огородов, рассчитать количество краски, необходимой для побелки потолка, сколько понадобится обоев для оклейки ко

мнаты и другое.

Геометрическая фигура

Для начала поговорим о прямоугольнике. Это фигура на плоскости, которая имеет четыре прямых угла, а ее противоположные стороны равны. Стороны его привыкли называть длиной и шириной. Измеряют их в миллиметрах, сантиметрах, дециметрах, метрах и т.

д. Теперь ответим на вопрос: «Как найти площадь прямоугольника?» Для этого необходимо длину умножить на ширину.

Площадь=длина*ширина

Но еще одна оговорка: длина и ширина должны быть выражены в одинаковых единицах измерения, то есть метр и метр, а не метр и сантиметр. Записывается площадь латинской буквой S. Для удобства обозначим длину латинской буквой b, а ширину латинской буквой a, как показано на рисунке. Отсюда мы делаем вывод, что единицей измерения площади является мм 2 , см 2 , м 2 и т. д.

Рассмотрим на конкретном примере, как найти площадь прямоугольника. Длина b=10 ед. Ширина a=6 ед. Решение: S=a*b, S=10 ед.*6 ед., S=60 ед 2 . Задача. Как узнать площадь прямоугольника, если длина в 2 раза больше ширины и составляет 18 м? Решение: если b=18 м, тогда а=b/2, a=9 м. Как найти площадь прямоугольника, если известны обе стороны? Правильно, подставить в формулу. S=a*b, S=18*9, S=162 м 2 . Ответ: 162 м 2 . Задача. Сколько необходимо купить рулонов обоев для комнаты, если ее размеры составляют: длина 5,5 м ширина 3,5, а высота 3 м? Размеры рулона обоев: длина 10 м, ширина 50 см.

Решение: сделаем рисунок комнаты.

Площади противоположных сторон равны. Вычислим площадь стены с размерами 5,5 м и 3 м. S стены 1 =5,5*3,

S стены 1 =16,5 м 2 . Следовательно, противоположная стена имеет площадь равную 16,5 м 2 . Найдем площади следующих двух стен. Стороны их, соответственно, равны 3,5 м и 3 м. S стены 2 =3,5*3, S стены 2 =10,5 м 2 . Значит, и противоположная сторона равна 10,5 м 2 . Сложим все результаты. 16,5+16,5+10,5+10,5=54 м 2 . Как вычислить площадь прямоугольника, если стороны выражены в разных единицах измерения. Ранее мы вычисляли площади в м 2 , то и в этом случае будем использовать метры. Тогда ширина рулона обоев будет равна 0,5 м. S рулона =10*0,5, S рулона =5 м 2 . Теперь узнаем, сколько рулонов необходимо для оклейки комнаты. 54:5=10,8 (рулонов). Так как они измеряются целыми числами, то нужно купить 11 рулонов обоев. Ответ: 11 рулонов обоев. Задача. Как вычислить площадь прямоугольника, если известно, что ширина на 3 см короче длины, а сумма сторон прямоугольника составляет 14 см? Решение: пусть длина х см, тогда ширина (х-3) см.

х+(х-3)+х+(х-3)=14, 4х-6=14, 4х=20, х=5 см — длина прямоугольника, 5-3=2 см — ширина прямоугольника, S=5*2, S=10 см 2 Ответ: 10 см 2 .

Резюме

Рассмотрев примеры, надеюсь, стало понятно, как найти площадь прямоугольника. Напомню, что единицы измерения длины и ширины должны совпадать, иначе получится неправильный результат, чтобы не допустить ошибок, читайте задание внимательно. Иногда сторона может быть выражена через другую сторону, не стоит бояться. Обратитесь к нашим решенным задачам, вполне возможно, они могут помочь. Но хоть раз в жизни мы сталкиваемся с нахождением площади прямоугольника.

Прямоугольник – это частный случай четырехугольника. Это значит, что у прямоугольника четыре стороны. Его противоположные стороны равны: так например, если одна из его сторон равна 10 см, то противоположная ей будет так же равны 10 см. Частным случаем прямоугольника является квадрат. Квадрат – это прямоугольник, у которого все стороны равны. Для вычисления площади квадрата можно пользоваться тем же алгоритмом, что и для вычисления площади прямоугольника. 2$.

Диагонали разделяет прямоугольник на 4 фигуры – 4 треугольника. При этом треугольники попарно равны. Если провести диагональ в прямоугольнике, то она разделяет фигуру на два равных прямоугольных треугольника. Средняя оценка: 4.4 . Всего получено оценок: 214.

Площадь геометрической фигуры — численная характеристика геометрической фигуры показывающая размер этой фигуры (части поверхности, ограниченной замкнутым контуром данной фигуры). Величина площади выражается числом заключающихся в нее квадратных единиц.

Формулы площади треугольника

  1. Формула площади треугольника по стороне и высоте
    Площадь треугольника равна половине произведения длины стороны треугольника на длину проведенной к этой стороне высоты
  2. Формула площади треугольника по трем сторонам и радиусу описанной окружности
  3. Формула площади треугольника по трем сторонам и радиусу вписанной окружности
    Площадь треугольника равна произведения полупериметра треугольника на радиус вписанной окружности.
  4. где S — площадь треугольника,
    — длины сторон треугольника,
    — высота треугольника,
    — угол между сторонами и,
    — радиус вписанной окружности,
    R — радиус описанной окружности,

Формулы площади квадрата

  1. Формула площади квадрата по длине стороны
    Площадь квадрата равна квадрату длины его стороны.
  2. Формула площади квадрата по длине диагонали
    Площадь квадрата равна половине квадрата длины его диагонали.
    S =1 2
    2
  3. где S — Площадь квадрата,
    — длина стороны квадрата,
    — длина диагонали квадрата.

Формула площади прямоугольника

    Площадь прямоугольника равна произведению длин двух его смежных сторон

    где S — Площадь прямоугольника,
    — длины сторон прямоугольника.

Формулы площади параллелограмма

  1. Формула площади параллелограмма по длине стороны и высоте
    Площадь параллелограмма
  2. Формула площади параллелограмма по двум сторонам и углу между ними
    Площадь параллелограмма равна произведению длин его сторон умноженному на синус угла между ними.

    a · b · sin α

  3. где S — Площадь параллелограмма,
    — длины сторон параллелограмма,
    — длина высоты параллелограмма,
    — угол между сторонами параллелограмма.

Формулы площади ромба

  1. Формула площади ромба по длине стороны и высоте
    Площадь ромба равна произведению длины его стороны и длины опущенной на эту сторону высоты.
  2. Формула площади ромба по длине стороны и углу
    Площадь ромба равна произведению квадрата длины его стороны и синуса угла между сторонами ромба.
  3. Формула площади ромба по длинам его диагоналей
    Площадь ромба равна половине произведению длин его диагоналей.
  4. где S — Площадь ромба,
    — длина стороны ромба,
    — длина высоты ромба,
    — угол между сторонами ромба,
    1 , 2 — длины диагоналей.

Формулы площади трапеции

  1. Формула Герона для трапеции

    Где S — Площадь трапеции,
    — длины основ трапеции,
    — длины боковых сторон трапеции,

Периметр квадрата и прямоугольника. Способы определения и примеры решения

Квадрат – это правильный четырехугольник, в котором все углы и стороны равны между собой.

Довольно часто эту фигуру рассматривают, как частный случай или . Диагонали квадрата равны между собой и используются в формуле площади квадрата через диагональ.
Для расчета площади рассмотрим формулу площади квадрата через диагонали:

То есть площадь квадрата равна квадрату длины диагонали поделенному на два. Учитывая, что стороны фигуры равны, можно рассчитать длину диагонали из формулы площади прямоугольного треугольника или по теореме Пифагора.

Рассмотрим пример расчета площади квадрата через диагональ. Пусть дан квадрат с диагональю d = 3 см. Необходимо вычислить его площадь:

По этому примеру расчета площади квадрата через диагонали мы получили результат 4,5 .

Площадь квадрата через сторону

Найти площадь правильного четырехугольника можно и по его стороне. Формула площади квадрата очень проста:

Так как в предыдущем примере расчета площади квадрата мы рассчитали значение по диаметру, теперь попробуем найти длину стороны:
Подставим значение в выражение:
Длина стороны квадрата будет равна 2,1 cm.

Очень просто можно использовать формулу площади квадрата вписанного в окружность.

Диаметр описанной окружности будет равен диаметру квадрата. Так как квадрат считается правильным ромбом, можно использовать формулу расчета площади ромба. Она равна половине произведения его диагоналей. Диагонали квадрата равны, значит формула будет выглядеть так:
Рассмотрим пример расчета площади квадрата вписанного в окружность.

Дан квадрат, вписанный в окружность. Диагональ окружности равна d = 6 см. Найдите площадь квадрата.
Мы помним, что диагональ окружности равна диагонали квадрата. Подставляем значение в формулу расчета площади квадрата через его диагонали:

Площадь квадрата равна 18

Площадь квадрата через периметр

В некоторых задачах по условиям дается периметр квадрата и требуется расчет его площади. Формула площади квадрата через периметр выводится из значения периметра. Периметр – это сумма длин всех сторон фигуры. Т.к. в квадрате 4 равных стороны, то он будет равенОтсюда находим сторону фигуры Площадь квадрата по обычной формуле считается так: .
Рассмотрим пример расчета площади квадрата через периметр.

Квадрат — это геометрическая фигура, представляющая собой четырехугольник все углы и стороны которого равны. Его также можно назвать прямоугольником , смежные стороны которого равны, или ромбом , у которого все углы равны 90º . Благодаря абсолютной симметрии найти площадь или периметр квадрата очень легко.

Инструкция:

  • Во-первых, определим, что периметром называется сумма длин всех сторон плоской геометрической фигуры, которая измеряется теми же величинами, что и длина. Вычислить периметр квадрата можно двумя способами.

Через длину стороны и диагонали

  • Поскольку периметр квадрата определяется суммой длин всех его сторон, а стороны у данной фигуры равны, то высчитать значение данной величины можно умножив длину одной стороны на число «4 ». Соответственно формулы будут выглядеть следующим образом: P = а + а + а + а или Р = а * 4 , где Р – это периметр квадрата и а длина стороны .
  • Кроме этого, в зависимости от условия задачи, периметр квадрата можно высчитать путем умножения длины его диагонали на два корня из двух: Р = 2√2 * d , где Р – это периметр квадрата и d — его диагональ .
  • Некоторые задачи требуют найти периметр квадрата , зная его площадь . Сделать это также не составит труда. Площадь данной фигуры равна длине его стороны, возведенной в квадрат: S = а 2 , где S площадь квадрата и а длина его стороны . Либо же площадь равна квадратному значению длины его диагонали, разделенному на два: S = d 2 /2 , где S – все та же площадь и d диагональ квадрата .
  • Зная формулы и значение площади, не трудно найти длину стороны или длину диагонали, а затем вернуться к формулам вычисления периметра и высчитать его значение.

Через радиус вписанной и описанной окружности

  • Наконец, немаловажно понимать и как найти периметр квадрата , если известен радиус окружности описанной вокруг него (или, напротив, в него вписанной). Вписанная в данную геометрическую фигуру окружность касается середины каждой стороны, и ее радиус равен половине любой стороны: R в = ½ а , где R в радиус вписанной окружности и а сторона квадрата .
  • Описанная окружность проходит через все вершины квадрата и ее радиус равен половине длины диагонали: R о = ½ d , где R о – это радиус описанной вокруг квадрата окружности и d – его диагональ .
  • Потому в первом случае периметр будет вычисляться по формуле: Р = 8 R в , а во втором: P = 4 х √2 х R о .

С помощью сайтов и интернет-калькулятора

  • Если вы вдруг по какой-то причине забыли формулы, то освежить знания поможет интернет. Зайдите в браузер, откройте страницу поисковика и в окне вбейте соответствующий запрос, например: «периметр квадрата формула ». Система выдаст огромное число сайтов справочного характера, которые помогут вам в данном вопросе, а также позволят справиться с решением задач, касающихся других геометрических фигур.
  • Кроме этого, если у вас нет желания разбираться в формулах и считать значения самостоятельно, то можно воспользоваться услугами интернет-калькуляторов . В качестве примера можно привести сайт . Раздел «Формулы периметра геометрических фигур » содержит теоретическую информацию, подкрепленную наглядными иллюстрациями. Если же перейти по ссылке «онлайн калькулятор », которая находится в окне каждой фигуры, то перед вами откроется страница для расчетов.
  • Выберите в окне снизу, на основании чего вы собираетесь высчитать периметр квадрата (сторона или диагональ), а затем введите имеющиеся данные. Система выдаст результат , руководствуясь установленными формулами.
  • Кроме этого, на сайте вы найдете и много другой информации, способной облегчить работу с математическими задачами . При желании можно поискать и более удобные или познавательные справочные сайты.
  • Если же вы не можете разобраться с самим ходом решения задачи, то здесь можно обратиться за помощью к людям, которые хорошо владеют методикой решения математических упражнений. Их всегда можно найти на соответствующих форумах , например,

Для вычисления площади и периметра квадрата нужно разобраться в понятиях этих величин. Квадрат представляет собой прямоугольник только с четырьмя одинаковыми сторонам, которые имеют между собой угол в 90°. Периметр — это сумма длин всех сторон. Площадь — это произведение длины прямоугольной фигуры на ее ширину.

Площадь квадрата и как ее найти

Как было сказано выше, квадрат — это прямоугольник, имеющий 4 равные стороны, поэтому ответом на вопрос: «как найти площадь квадрата» является формула: S = a*a или S = a 2 , где а — сторона квадрата. Исходя из этой формулы, легко находится сторона квадрата, если известна площадь. Для этого необходимо извлечь квадрат из указанной величины.

Например, S = 121, следовательно, а = √121 = 11. Если заданное значение отсутствует в таблице квадратов, то можно воспользоваться калькулятором: S = 94, а = √94 = 9,7.

Как найти периметр квадрата

Периметр квадрата находится по легкой формуле: Р = 4а, где а — сторона квадрата.

Пример:

  • сторона квадрата = 5, следовательно, P = 4*5 = 20
  • сторона квадрата = 3, следовательно, Р = 4*3 = 12

Но существуют такие задачи, где заведомо обозначена площадь, а нужно найти периметр. При решении нужны формулы, которые представлены ранее.

Например: как найти периметр квадрата, если известна площадь, равная 144?

Шаги решения:

  1. Выясняем длину одной стороны: а = √144 = 12
  2. Находим периметр: Р = 4*12 = 48.

Нахождение периметра вписанного квадрата


Существуют еще несколько способов нахождения периметра квадрата. Рассмотрим один из них: нахождение периметра через радиус описанной окружности. Здесь появляется новый термин «вписанный квадрат» — это квадрат, чьи вершины лежат на окружности.

Алгоритм решения:


  • так как на рассмотрении квадрат, формулу можно выразить таким образом: a 2 + a 2 = (2r) 2 ;
  • затем следует уравнение сделать проще: 2a 2 = 4(r) 2 ;
  • делим уравнение на 2: (a 2 ) = 2(r) 2 ;
  • извлекаем корень: a = √(2r).

В итоге получаем последнюю формулу: а (сторона квадрата) = √(2r).

  1. Найденная сторона квадрата умножается на 4, далее применяется стандартная формула по нахождению периметра: P = 4√(2r).

Задача:

Дан квадрат, который вписан в окружность, ее радиус равен 5. Значит, диагональ квадрата равняется 10. Применяем теорему Пифагора: 2(a 2 ) = 10 2 , то есть 2a 2 = 100. Делим полученное на два и в результате: a 2 = 50. Так как это не табличное значение, используем калькулятор: а = √50 = 7,07. Умножаем на 4: Р = 4*7,07 = 28,2. Задача решена!

Рассмотрим еще один вопрос

Часто в задачах встречается другое условие: как найти площадь квадрата, если известен периметр?

Мы уже рассмотрели все необходимые формулы, поэтому для решения задач подобного типа, необходимо умело их применять и связывать между собой. Перейдем сразу к наглядному примеру: Площадь квадрата равна 25 см 2 , найдите его периметр.

Шаги решения:

  1. Находим сторону квадрата: а = √25 = 5.
  1. Находим сам периметр: Р = 4*а = 4*5 = 20.

Подводя итог, важно напомнить, что такие легкие формулы применимы не только в учебной деятельности, но и повседневной жизни. Периметр и площадь фигуры дети учатся находить еще в начальной школе. В средних классах появляется новый предмет — геометрия, где теорема Пифагора находится в самом начале изучения. Эти азы математики проверяются и по окончанию школы ОГЭ и ЕГЭ, поэтому важно знать эти формулы и правильно их применять.

Содержимое:

Периметр двумерной фигуры – это общая длина ее границы, равная сумме длин сторон фигуры. Квадрат – это фигура с четырьмя сторонами одинаковой длины, которые пересекаются под углом 90°. Так как в квадрате все стороны имеют одинаковую длину, то вычислить его периметр очень легко. Эта статья расскажет вам, как вычислить периметр квадрата по одной данной стороне, по данной площади и по данному радиусу окружности, описанной вокруг квадрата.

Шаги

1 Вычисление периметра по данной стороне

  1. 1 Формула для вычисления периметра квадрата: P = 4s , где s – длина стороны квадрата.
  2. 2 Определите длину одной стороны квадрата и умножьте ее на 4, чтобы найти периметр. Чтобы определить длину стороны, измерьте ее линейкой или посмотрите ее значение в учебнике (задаче). Вот некоторые примеры вычисления периметра:
    • Если сторона квадрата равна 4, то P = 4 * 4 = 16 .
    • Если сторона квадрата равна 6, то P = 4 * 6 = 36 .

2 Вычисление периметра по данной площади

  1. 1 Формула для вычисления площади квадрата. Площадь любого прямоугольника (а квадрат – это частный случай прямоугольника) равна произведению его длины на его ширину. Поскольку длина и ширина квадрата равны, то его площадь вычисляется по формуле: A = s*s = s 2 , где s – длина стороны квадрата.
  2. 2 Извлеките квадратный корень из значения площади, чтобы найти сторону квадрата. Для этого в большинстве случаев воспользуйтесь калькулятором (введите значение площади и нажмите клавишу «√»). Вы также можете вычислить.
    • Если площадь квадрата равна 20, то его сторона равна: s = √20 = 4,472 .
    • Если площадь квадрата равна 25, то s = √25 = 5 .
  3. 3 Умножьте найденную сторону на 4, чтобы найти периметр. Вычисленное значение стороны подставьте в формулу для нахождения периметра: P = 4s . Вы найдете периметр квадрата.
    • В нашем первом примере: P = 4 * 4,472 = 17,888 .
    • Периметр квадрата, площадь которого равна 25, а сторона равна 5, равен Р = 4 * 5 = 20 .

3 Вычисление периметра по данному радиусу окружности, описанной вокруг квадрата

  1. 1 Вписанный квадрат – это квадрат, вершины которого лежат на окружности.
  2. 2 Отношение между радиусом окружности и длиной стороны квадрата. Расстояние от центра описанной окружности до вершины вписанного в нее квадрата равно радиусу окружности. Чтобы найти сторону квадрата s , необходимо диагональю разделить квадрат на 2 прямоугольных треугольника. Каждый из этих треугольников будет иметь равные стороны a и b и общую гипотенузу с , равную удвоенному радиусу описанной окружности (2r ).
  3. 3 Воспользуйтесь теоремой Пифагора, чтобы найти сторону квадрата. Теорема Пифагора гласит, что в любом прямоугольном треугольнике с катетами а и b и гипотенузой с : a 2 + b 2 = c 2 . Так как в нашем случае а = b (не забывайте, что мы рассматриваем квадрат!), и мы знаем, что с = 2r , то мы можем переписать и упростить это уравнение:
    • a 2 + a 2 = (2r) 2 «»; теперь упростим это уравнение:
    • 2a 2 = 4(r) 2 ; теперь разделим обе стороны уравнения на 2:
    • (a 2) = 2(r) 2 ; теперь извлечем квадратный корень из обеих сторон уравнения:
    • a = √(2r) . Таким образом, s = √(2r) .
  4. 4 Умножьте найденную сторону квадрата на 4, чтобы найти его периметр. В этом случае периметр квадрата: P = 4√(2r) . Эту формулу можно переписать так: Р = 4√2 * 4√r = 5,657r , где r – радиус описанной окружности.
  5. 5 Пример. Рассмотрим квадрат, вписанный в окружность радиусом 10. Это означает, что диагональ квадрата равна 2 * 10 = 20. Используя теорему Пифагора, мы получим: 2(a 2) = 20 2 , то есть 2a 2 = 400. Теперь разделим обе стороны уравнения на 2 и получим: a 2 = 200. Теперь извлечем квадратный корень из обеих сторон уравнения и получим: а = 14,142 . Умножим это значение на 4 и вычислим периметр квадрата: P = 56,57 .
    • Обратите внимание, что вы могли бы получить тот же результат, просто умножив радиус (10) на 5,657: 10 * 5,567 = 56,57 ; но такой метод трудно запомнить, поэтому лучше пользоваться процессом вычисления, описанным выше.

§21. Площадь. Площадь прямоугольника (Страницы 87,88,89,90,91,92)

ПОВТОРЯЕМ ТЕОРИЮ

243. Заполните пропуски.

1) Равные фигуры имеют равные площади.
2) Площадь фигуры равна сумме площадей фигур, из которых она состоит.
3) За единицу измерения площади выбирают квадрат, сторона которого равна единичному отрезку.
4) Измерить площадь фигуры — значит подсчитать, сколько единичных квадратов в ней помещается.
5) 1 см2 — это площадь квадрата со стороной 1 см.
6) 1 дм2 — это площадь квадрата со стороной 1 дм.
7) Площадь прямоугольника вычисляют по формуле S=a*b, где S — его площадь, a и b — длины соседних сторон, выраженные в одних и тех же единицах.
8) Площадь квадрата вычисляют по формуле S=a2, где S — его площадь, a — его сторона.
9) 1 м2 = 10000 см2
10) 1 км2 = 1000000 м2
11) 1 а = 100 м2
12) 1 га = 10000 м2 = 100 а

РЕШАЕМ ЗАДАЧИ

244. Если стороны прмоугольника равны 12 см и 8 см, то его площадь

S = 12*8 = 96 см2

245. Если сторона квадрата равна 9 дм, то его площадь

S = 92 = 81 дм2

246. Заполните пропуски.

1) 6 а = 600 м2
12 га = 120000 м2
3 га 42 а = 34200м2

2) 7 га = 700 а
6 га 5 а = 605 а
72 000 м2 = 720 а

3) 4 дм2 = 400 см2
4 м2 = 40000 см2
2 м2 35 дм2 = 23500 см2

4) 270000 м2 = 27 га
8000 а = 80 га
2 км2 = 200 га

247. Сравните величины.

248. Заполните таблицу, где S — площадь прямоугольника, a и b — длины его соседних сторон.

а3 дм8 дм40 см5 км36 см30 м
b6 см5 дм9 дм4 м6 дм4 км
S180 см240 дм236 дм220 га2160 см212 а

1) 30 см*6 см = 180 см2
2) 8 дм*5 дм = 40 дм2
3) 40 см (4 дм)*9 дм = 36 дм2
4) 5 км (5000 м)*4 м = 20000 км = 20 га
5) 36 см*6 дм (60 см) = 2160 см2
6) 30 м*4 км(4000 м) = 120000 м = 12 а

249. Найдите площадь квадрата, периметр которого равен 64 см.

Решение:
1) 64:4 = 16 (см) длина сторон квадрата
2) 16*16 = 256 (см2) площадь квадрата

Ответ: Площадь квадрата 256 см2.

250. Поле прмоугольной формы имеет площадь 42 а, его длина равна 70 м. Вычислите периметр поля.

Решение:
42 а = 42000 м2
1) 42000:70 = 600 (м) ширина поля
2) 2*(70*600) = 2600 (м) периметр поля

Ответ: периметр поля равен 2600 м.

251. На рисунке изображен прямоугольник ABCD, у которого АD=8 см, АВ=4 см. Точка К — середина отрезка АD, точка М — середина отрезка АК, точка F — середина отрезка АВ, точка Е — середина отрезка АF. Чему равна площадь закрашенного прямоугольника?

Решение:
(8:4)*(4:4) = 2*1 = 2 (см2)

Ответ: Площадь закрашенного прямоугольника равна 2 см2.

252. На рисунке изображены три квадрата. Середины сторон большого квадрата являются вершинами среднего квадрата, а середины сторон среднего квадрата — вершинами маленького квадрата. Площадь маленького квадрата равна 25 см2. Чему равна площадь квадрата?

Решение:
25*4 = 100 (см2)

Ответ: площадь квадрата равна 100 см2

253. Вычислите площадь фигуры, изображенной на рисунке (размеры даны в сантиметрах).

Решение:
(12+3)*6-(2*2*3+2*2) = 74 (см2) или
12*6+2*3-2*2 = 74 (см2)

Ответ: площадь фигуры равна 74 см2.

254. Заполните цепочку вычислений.

1) 600 а : 300 = 2 а
2) 2 а + 4 а = 6 а
3) 600 м2 — 120 м2 = 480 м2
4) 48000 дм2 : 800 = 60 дм2
5) 60 дм2 — 28 дм2 = 32 дм2

255. Сколько надо рулонов обоев, чтобы оклеить ими стену длиной 7 м и высотой 4 м, если длина рулона равна 10 м, а ширина — 50 см?

Решение:
1) 7*4 = 28 (м2) площадь стены
2) 50*1000 = 50000 (см2) = 5 (м2) содержит улон обоев
3) 28:5 = 5 ост.3 (рулоны)

Ответ: 6 рулонов

256. С огорода, который имеет форму приямоугольника со сторонами 50 м и 30 м, собрали 180 ведер картофеля. В одно ведро помещается 8 кг картофеля. Сколько килограммов картофеля собрали с 1 а?

Решение:
1) 50*30 = 1500 (м2) = 15 а — площадь огорода.
2) 180:15 = 12 (ведер) с одного а
3) 12*8 = 96 (кг) с одного а

Ответ: 96 кг с одного а.

257. Длина прямоугольника равна 28 см. На сколько квадратных сантиметров увеличится его площадь, если ширину этого прямоугольника увеличить на 3 см?

Решение:
28*(х+3)-28*х = 28х+28*3-28х = 28*3 = 84 (см2)

Ответ: площадь увеличится на 84 см2.

258. Во сколько раз увеличится периметр и во сколько раз увеличится площадь прямоугольника, если каждую его сторону увеличить в 3 раза?

Решение:
1) 2*(3а+3b):2(а+b) = 6а+6b:2а+2b = 6(а+b):2(а+b) = 6:2 = 3
2) (3а*3b):(а*b) = 9аb аb = 9

Ответ: в 3 раза; в 9 раз.

259. На рисунке изображен квадрат, разбитый на шесть прямоугольников, сумма периметров которых равна 80 см. Чему равна площадь квадрата?

Решение:
4а+4а = 80
8а = 80
а = 10 (см)

S = а2 = 102 = 100 (см2)

Ответ: площадь квадрата равна 100 см2.

Оглавление

§20. Степень числа — стр. 85-87§22. Прямоугольный параллелепипед. Пирамида — стр. 93-99

Сохраните или поделитесь с одноклассниками:

Длина стороны квадрата равна 6 см .его площадь равна площади прямоугольника,

Математика, 2021-03-02 22:35:40, сел5

Ответ

Ответ разместил: getmanev

Площадь равна 6*6=36см
Ширина равна 36/9=4 см

Ответ

Ответ разместил: galkaoks056

1) 6*6 = 36 (см2) — площадь квадрата.
2) 36 : 9 = 4 (см) — ширина прямоугольника.

Ответ

Ответ разместил: nikfyodorov20

1) 6*6=36 (см) — площадь квадрата и прямоугольника
2) 36:9=4 (см)
ответ: ширина прямоугольника 4 см

Ответ

Ответ разместил: mintotea

1) 6*6=36см площадь квадрата
2) 6+6=12 длина двух сторон прямоугольника
3)36-12=24см длина других 2 сторон прямоугольника
24:2=12 см
ответ 12см

Ответ

Ответ разместил: ilmir123456789

1) 9×6=54(см)– площ. прямоугольника.
2)54÷9=6(см)–ширина прямоугольника.

Ответ

Ответ разместил: okszav

1)6*6=36 (см) в квадрате-площадь квадрата
2)36/9=4 (см)-ширина прямоугольника 
ответ:4 см ширина прямоугольника
Поставь и лучший ответ это правильный ответ

Ответ

Ответ разместил: abakanilaha

1)6*6=36(см.) 2)36:9=4(см). ответ: 4 см ширина прямоугольника

Ответ

Ответ разместил: SolekProAwp

6 см — сторона квадрата
9 см — длина прямоугольника
Sкв. =Sпр.
ширина прямоуг. — ? см

1)   6*6=36(см²) — S квадрата
2)   36:9=4(см) — ширина прямоугольника

Ответ

Ответ разместил: mutsienietse14

6*6 равно 36 площадь квадрата . 36:9 равно 4- это ширина прямоугольника.

Ответ

Ответ разместил: 10012011

6*6=36 см2 — площадь квадрата и площадь прямоугольника
36/9=4 см — ширина прямоугольника

Другие вопросы по: Математика

.(Найти три числа, сумма которых равна 300,если второе число в 2 раза больше первого, а третье в 3 раза больше первого)….

Опубликовано: 26.02.2019 13:00

Ответов: 1

.(Длина прямоугольного параллелепипеда равна 5 дм, ширина на 6 см меньше длины, а высота составляет 5/7 ширины . найди объём и площадь поверхности этого прямоугольного параллелепип…

Опубликовано: 28.02.2019 00:10

Ответов: 1

Сколькими футбольных команд могут разыграть между собой золотые, бронзовые и серебряные медали?. ..

Опубликовано: 28.02.2019 06:00

Ответов: 1

Найдите электрическое сопротивление грифеля диаметром 2 мм в карандаше стандартной длины 17.5 см. удельное сопротивление графита равно 13 ом*мм2\м…

Опубликовано: 01.03.2019 10:00

Ответов: 3

Вчетырех танцевальных студиях 172 ученика. в первой студии в 4 раза меньше чем во всех четырех; во второй на 8 человек меньше , чем в первой, в остальных двух поровну. сколько учен…

Опубликовано: 02.03.2019 15:00

Ответов: 3

При каких значениях x дробь 8-xбудет неправельной…

Опубликовано: 02.03.2019 15:40

Ответов: 3

Популярные вопросы

.(Введите здесь ваше двигатель мощностью 20 квт совершил 12 мдж работы. сколько времени он работал?)….

Опубликовано: 27.02.2019 15:40

Ответов: 2

Что тако обмен веществ вашими словами? ?. ..

Опубликовано: 28.02.2019 05:50

Ответов: 1

Высота правильной прямоугольной пирамиды равна h, а двугранный угол при стороне основания равен 45(градусам). найти площадь поверхности пирамиды….

Опубликовано: 28.02.2019 08:00

Ответов: 2

Определить истинно или ложно: вода с сахором это соединение вода это чистое вещество испарение жидкости с последующим охладением и конденсацией называется дистилляция….

Опубликовано: 01.03.2019 23:00

Ответов: 1

Какова траектория конца минутной стрелки…

Опубликовано: 02.03.2019 16:40

Ответов: 1

Поставь знак ударения повидло, веселый, лисята, яйцо, сугробы, помещение, царица, шлем, цыпленок, малиновка, выглянуло, певица, черные, выдумщик, экскаватор, клен, сладкий, два, уй…

Опубликовано: 03.03.2019 11:00

Ответов: 3

Снимите наружные сухие чешуи, установите, что расположено под ними. разрежте луковицу вдоль. сравните внутренние чешуи с наружными. каково значение этих видоизмененных листьев?…

Опубликовано: 03.03.2019 17:30

Ответов: 2

Ск-ко существует двузначных чисел в записи которых обе цифры чётные?…

Опубликовано: 04.03.2019 01:00

Ответов: 1

Втрапеции авсд угол а=90, ас=6, вс=6 корней из 2х, де-высота треугольника асд, а tg угла асд=2. найти се….

Опубликовано: 04.03.2019 01:10

Ответов: 3

Что недопустимо в предпринимательстве? 7-10 пунктов…

Опубликовано: 04.03.2019 01:40

Ответов: 1

Больше вопросов по предмету: Математика Случайные вопросы

Задачи про площадь квадрата и многое другое

Такой удивительный и знакомый квадрат. Он симметричен относительно своего центра и осей, проведенных по диагоналям и через центры сторон. А искать площадь квадрата или его объем вообще не составляет большого труда. Особенно если известна длина его стороны.

Несколько слов о фигуре и ее свойствах

Первые два свойства связаны с определением. Все стороны фигуры равны друг другу. Ведь квадрат — это правильный четырехугольник. Причем у него обязательно все стороны равны и углы имеют одинаковое значение, а именно — 90 градусов. Это второе свойство.

Третье связано с длиной диагоналей. Они тоже оказываются равными друг другу. Причем пересекаются под прямыми углами и в точках середины.

Формула, в которой используется только длина стороны

Сначала об обозначении. Для длины стороны принято выбирать букву «а». Тогда площадь квадрата вычисляется по формуле: S = а2.

Она легко получается из той, что известна для прямоугольника. В ней длина и ширина перемножаются. У квадрата эти два элемента оказываются равными. Поэтому в формуле появляется квадрат этой одной величины.

Формула, в которой фигурирует длина диагонали

Она является гипотенузой в треугольнике, катетами которого являются стороны фигуры. Поэтому можно воспользоваться формулой теоремы Пифагора и вывести равенство, в котором сторона выражена через диагональ.

Проведя такие несложные преобразования, получаем, что площадь квадрата через диагональ вычисляется по такой формуле:

S = d

2 / 2. Здесь буквой d обозначена диагональ квадрата.

Формула по периметру

В такой ситуации необходимо выразить сторону через периметр и подставить его в формулу площади. Поскольку одинаковых сторон у фигуры четыре, то периметр придется разделить на 4. Это будет значение стороны, которую потом можно подставить в начальную и сосчитать площадь квадрата.

Формула в общем виде выглядит так: S = (Р/4)2.

Задачи на расчеты

№ 1. Имеется квадрат. Сумма двух его сторон равна 12 см. Вычислите площадь квадрата и его периметр.

Решение. Поскольку дана сумма двух сторон, то нужно узнать длину одной. Так как они одинаковые, то известное число нужно просто разделить на два. То есть сторона данной фигуры равна 6 см.

Тогда его периметр и площадь легко вычисляются по приведенным формулам. Первый равен 24 см, а вторая — 36 см2.

Ответ. Периметр квадрата равняется 24 см, а его площадь — 36 см2.

№ 2. Узнайте площадь квадрата с периметром, равным 32 мм.

Решение. Достаточно просто подставить значение периметра в написанную выше формулу. Хотя можно сначала узнать сторону квадрата, а уже потом его площадь.

В обоих случаях в действиях сначала будет идти деление, а потом возведение в степень. Простые расчеты приводят к тому, что площадь представленного квадрата равна 64 мм2.

Ответ. Искомая площадь равна 64 мм2.

№ 3. Сторона квадрата равна 4 дм. Размеры прямоугольника: 2 и 6 дм. У какой из этих двух фигур больше площадь? На сколько?

Решение. Пусть сторона квадрата будет обозначена буквой а1, тогда длина и ширина прямоугольника а2 и в2. Для определения площади квадрата значение а1 полагается возвести в квадрат, а прямоугольника — перемножить а2 и в2 . Это несложно.

Получается, что площадь квадрата равна 16 дм2, а прямоугольника — 12 дм2. Очевидно, что первая фигура больше второй. Это при том, что они равновелики, то есть имеют одинаковый периметр. Для проверки можно сосчитать периметры. У квадрата сторону нужно умножить на 4, получится 16 дм. У прямоугольника сложить стороны и умножить на 2. Будет то же число.

В задаче требуется еще ответить, на сколько площади различаются. Для этого из большего числа вычитают меньшее. Разница оказывается равной 4 дм2.

Ответ. Площади равны 16 дм2 и 12 дм2. У квадрата она больше на 4 дм2.

Задача на доказательство

Условие. На катете равнобедренного прямоугольного треугольника построен квадрат. К его гипотенузе построена высота, на которой построен еще один квадрат. Доказать, что площадь первого в два раза больше, чем второго.

Решение. Введем обозначения. Пусть катет равен а, а высота, проведенная к гипотенузе, х. Площадь первого квадрата — S1, второго — S2.

Площадь квадрата, построенного на катете, вычисляется просто. Она оказывается равной а2. Со вторым значением все не так просто.

Для начала нужно узнать длину гипотенузы. Для этого пригодится формула теоремы Пифагора. Простые преобразования приводят к такому выражению: а√2.

Поскольку высота в равнобедренном треугольнике, проведенная к основанию, является еще и медианой и высотой, то она делит большой треугольник на два равных равнобедренных прямоугольных треугольника. Поэтому высота равна половине гипотенузы. То есть х = (а√2)/2. Отсюда легко узнать площадь S2. Она получается равной а2/2.

Очевидно, что записанные значения отличаются ровно в два раза. Причем вторая в это число раз меньше. Что и требовалось доказать.

Необычная головоломка — танграм

Она делается из квадрата. Его необходимо по определенным правилам разрезать на различные фигуры. Всего частей должно оказаться 7.

Правила предполагают, что в процессе игры будут использоваться все получившиеся детали. Из них нужно составлять другие геометрические фигуры. Например, прямоугольник, трапецию или параллелограмм.

Но еще интереснее, когда из кусочков получаются силуэты животных или предметов. Причем оказывается, что площадь всех производных фигур равна той, что была у начального квадрата.

Как найти длину стороны квадрата

Все ресурсы по базовой геометрии

9 Диагностические тесты 164 практических теста Вопрос дня Карточки Learn by Concept

← Предыдущая 1 2 3 4 5 6 Следующая →

Справка по базовой геометрии » Плоская геометрия » Четырехугольники » Площади » Как найти длину стороны квадрата

Квадрат имеет площадь , какова длина его стороны?

Возможные ответы:

Правильный ответ:

Пояснение:

Стороны можно найти, извлекая квадратный корень из площади.

, где = сторона.

.

Значит, длина стороны 6,3 см.

Сообщить об ошибке

Периметр квадрата равен 16. Найдите длину каждой стороны этого квадрата.

Возможные ответы:

4

3

8

4

12

Правильный ответ:

4

Пояснение:

Во-первых, знайте, что все длины сторон квадрата равны. Во-вторых, знайте, что сумма длин всех четырех сторон дает нам периметр. Таким образом, периметр квадрата числа 16 записывается как

, где S — длина стороны квадрата. Решите для этого S

Таким образом, длина каждой стороны этого квадрата равна 4.

Сообщить об ошибке

Детская площадка огорожена квадратным забором. Площадь детской площадки составляет . Периметр забора составляет . Какова длина одной стороны забора?

 

Возможные ответы:

Правильный ответ:

Объяснение:

У нас будет две формулы, которые помогут нам решить эту задачу: площадь и периметр квадрата.

Площадь квадрата:

 ,

, где длина квадрата и ширина квадрата.

Периметр квадрата:

Подставив наши значения, мы получим:

Поскольку все стороны квадрата имеют одинаковое значение, мы можем заменить все и на (сторона). Наши уравнения принимают вид:

 

 

Следовательно, .

Сообщить об ошибке

Площадь показанного ниже квадрата составляет 36 квадратных дюймов. Какова длина одной из сторон?

Возможные ответы:

Невозможно определить из предоставленной информации.

Правильный ответ:

Пояснение:

Площадь любого четырехугольника можно определить, умножив длину его основания на высоту.

Поскольку мы знаем, что фигура здесь квадратная, мы знаем, что все стороны имеют одинаковую длину. Исходя из этого, мы можем работать в обратном направлении, взяв квадратный корень из площади, чтобы найти длину одной стороны.

Длина одной (и каждой) стороны этого квадрата равна 6 дюймов.

Сообщить об ошибке

У квадрата одна сторона длины , какова длина противоположной стороны?

Возможные ответы:

Правильный ответ:

Пояснение:

Одним из необходимых условий квадрата является то, что все стороны должны быть одинаковой длины. Следовательно, поскольку нам дана длина одной стороны, мы знаем длину всех сторон, включая длину противоположной стороны. Поскольку длина одной из сторон равна 4, мы можем заключить, что все стороны равны 4, а это означает, что длина противоположной стороны равна 4.

Сообщить об ошибке

Периметр квадрата равен половине его площади. Какова длина одной стороны квадрата?

Возможные ответы:

Правильный ответ:

Пояснение:

Начнем с формул для периметра и площади квадрата соответственно.

Используя эти формулы и тот факт, что периметр равен половине площади, мы можем составить уравнение.

Мы можем умножить обе части на 2, чтобы исключить дробь.

Чтобы одна часть уравнения была равна нулю, переместим все в правую часть.

Далее мы можем факторизовать.

Установка каждого коэффициента равным нулю обеспечивает два возможных решения.

       или        

                                               

     

Сообщить об ошибке

Если площадь квадрата составляет 100 квадратных единиц, какова в единицах длина одной стороны квадрата?

Возможные ответы:

Правильный ответ:

Объяснение:

Сообщить об ошибке

In Square , . Оцените  с точки зрения .

Возможные ответы:

Правильный ответ:

Объяснение:

Если построить диагональ Квадрата, то это треугольник 45-45-90 с гипотенузой. По теореме 45-45-90 длину стороны можно вычислить следующим образом:

.

Сообщить об ошибке

Окружность, описанная вокруг квадрата, имеет длину окружности 20. Оценить с точностью до десятых.

Возможные ответы:

Правильный ответ:

Объяснение:

Диаметр круга с длиной окружности 20 равен

Диаметр круга, описывающего квадрат, равен длине диагоналей квадрата.

Если построить диагональ Квадрата, то получится треугольник 45-45-90 с гипотенузой приблизительно 6,3662. По теореме 45-45-90 разделите это на , чтобы получить длину стороны квадрата:

Сообщить об ошибке

Прямоугольник имеет площадь, равную 90% площади квадрата, и составляет 80% площади. Какой процент от ?

Возможные ответы:

Правильный ответ:

Объяснение:

Площадь квадрата  – это квадрат длины стороны или .

Площадь прямоугольника равна . Прямоугольник  имеет площадь, равную 90 % площади Квадрата , то есть ; составляет 80% от , поэтому . Мы можем составить следующее уравнение:

В процентах от

Сообщить об ошибке

← Предыдущий 1 2 3 4 5 6 Следующий →

Уведомление об авторских правах

Все ресурсы по базовой геометрии

9 Диагностические тесты 164 практических теста Вопрос дня Карточки Learn by Concept

задач с прямоугольниками

задач с прямоугольниками

Математический сайт миссис Нафольц

Задачи с прямоугольниками

Если сторону квадрата увеличить на 3 м, его площадь составит 121 м 2 . Найдите длину стороны исходного квадрата.

(s + 3) 2 = 121
с 2 + 6s + 9 = 121
с 2 + 6s — 112 = 0
(s + 14)(s — 8) = 0
с = 8 is единственный разумный ответ.

Длина прямоугольного стола на 5 дюймов больше его ширины более чем в два раза. Его площадь 1950 в 2 . Найдите его размеры.

Пусть w = ширина.
Длина = 2w + 5
w(2w + 5) = 1950
2w 2 + 5w — 1950 = 0
(2w + 65)(w — 30) = 0
w = 30 является единственным разумным решением.
ОТВЕТ: 30 на 65.

Диагональ прямоугольника 25 см, а длина 24 см. Найдите ширину прямоугольника.

Используя теорему Pythagorean Мы получаем:
L 2 + W 2 = DIAG 2
24 2 + W 2 = 25 2
5 576 + 2 = 25 2
555 576 + 2 = 25 . = 49
ш = 7

Длина фотографии на 12 см больше ее ширины. Он вмонтирован в рамку шириной 5 см. Площадь рамы 620 см 2 . Найдите размеры фотографии.

 

Length

Width

Area

Photo

n + 12

n

n 2 + 12n

Рамка Photo plus

n + 22

n + 10

n 2 + 32н + 220

Площадь рамы = 620.
n 2 + 32n + 220 — (n 2 + 12n) = 620
n 2 +32n + 220 — n 2 — 5 202 — 12n = 4 620 12n = 620
20n + 220 = 620
20n = 400
n = 20
Ответ: Размеры фотографии 20 на 32.

Сторона квадрата имеет длину s. Длина прямоугольника на 4 м больше стороны квадрата, а ширина прямоугольника на 2 м меньше стороны квадрата. Периметр прямоугольника на 24 м меньше удвоенного периметра квадрата. Найдите размеры каждой фигуры.

 

Length

Width

Perimeter

Square

n

n

4n

Rectangle

н + 4

н — 2

4н + 4

Периметр прямоугольника на 24 меньше, чем удвоенный периметр квадрата.
4n + 4 = 2(4n) — 24
4n + 4 = 8n — 24
4n + 28 = 8n
28 = 4n
n = 7
Ответ: Квадрат 7 на 7. Прямоугольник 11 на 5.

Длина прямоугольника на 1 см меньше его ширины в три раза. Если площадь прямоугольника
44 см 2 , найдите размеры.

Ширина = w
Длина = 3w — 1
Площадь = 44
w(3w — 1) = 44
3w 2 — w = 44
3w 2 — w — 44 = 0
(3w + 11)(w — 4) = 0
3w + 11 = 0 ИЛИ w — 4 = 0
w = -11/3 НЕ ДЕЙСТВИТЕЛЬНЫЙ РАЗМЕР ИЛИ w = 4.
Ответ: Ширина = 4, Длина = 11

Прямоугольное ограждение с одной перегородкой должно быть ограничено стеной и 30-метровым ограждением, как показано на схеме. Какими должны быть размеры прямоугольника площадью 48 м 2

Забор состоит из 4 частей: 3 параллельных секций, каждая длиной x, и 1 секция, параллельная стене, длина которой должна быть 30 — 3x.
Размеры шкафа: ширина = x, длина = 30 — 3x.
Площадь = 48.
x (30 — 3x) = 48
30x — 3x 2 = 48    Прибавьте 3x 2 и вычтите 30x
0 = 3x 2 — 3 903 F + 48 out. (x 2 — 10x + 16)
0 = 3(x — 2)(x — 8)
x — 2 = 0 ИЛИ x = 8 = 0.
x = 2 ИЛИ x = 8.
Если x = 2, тогда длина = 30 — 3(2) = 24. Ответ: 2 на 24
Если x = 8, то длина = 30 — 3(8) = 6. Ответ: 8 на 6.
Обе конфигурации допустимы.

Длина прямоугольника на 6 м больше его ширины более чем в 2 раза. Если площадь прямоугольника 140 м 2 , найти размеры.

Размеры прямоугольника равны w и 2w + 6,
. Площадь = 140,
. 2 + 3w — 70 = 0
2(w + 10)(w — 7) = 0
w + 10 = 0 ИЛИ w — 7 = 0
w = -10 Неверный размер. ИЛИ w = 7.
Ответ: ширина = 7 и длина = 2(7) + 6 = 20.

Длина прямоугольного бассейна больше его ширины на 8 м. Бассейн окружает дорожка шириной 2 м. Каковы размеры бассейна, если площадь дорожки 176 м 2 ?

2 + 16N + 96

902 9029

0415

 

Length

Width

Area

Pool

n + 8

n

n 2 + 8n

Бассейн и проход

N + 12

N + 4

N 2 + 16N + 96

Площадь прохода = 176.
n 2 + 16n + 96 — (n 2 + 8n) = 176
n 2 + 16n + 96 — n 2 + 6 — 8n = 359 176 8n = 176
8n + 96 = 176
8n = 80
n = 10
Ответ: Размеры бассейна 10 на 18.

Длина садового участка прямоугольной формы на 10 м больше его ширины. Если длину увеличить на 1 м, а ширину уменьшить на 2 м, площадь уменьшится на 42 м 2 . Какие исходные размеры?

Новая площадь на 42 меньше старой.
(w + 11)(w — 2) = w(w + 10) — 42
w 2 -2w + 11w — 22 = w 2 + 10w — 42
9w — 22 = 10w — 42  Добавить — 10w
-w — 22 = -42  Добавить 22
-w = -20
w = 20
Ответ: исходные размеры были 20 на 30

.

Высота плаката на 25 см больше его ширины. Его монтируют на кусок картона так, чтобы со всех сторон был бордюр в 5 см. Если площадь одной только границы 1350 см 2 , какие размеры плаката?

Внешняя площадь — Внутренняя площадь = Граница = 1350
(x + 35)(x + 10) — x(x + 25) = 1350
x 2 + 45x + 350 — x 2 — 25x = 1350
20x + 350 = 1350
20x = 1000
x = 50
ОТВЕТ: 50 на 75

Длина прямоугольника вдвое больше, чем его ширина. Если его длину и ширину уменьшить на 4 см, его площадь уменьшится на 164 см 2 . Найдите его первоначальные размеры.

Новая площадь на 164 меньше старой.
(2w — 4)(w — 4) = 2w 2 — 164
2w 2 — 8w — 4w + 16 = 2w 2 — 164
-12w + 16 = -164    3 = -180
w = 15
Ответ: Исходные размеры: 15 см на 30 см.

Длина прямоугольника в три раза больше его ширины. Если его длину и ширину увеличить на 3 м, его площадь увеличится на 81 м 2 . Найдите его первоначальные размеры.

Новый район на 81 больше старого.
(3w + 3)(w + 3) = 3w 2 + 81
3w 2 + 9w + 3w + 9 = 3w 2 + 81
12w + 9 = 81    Добавить -9
12w = 72
w = 6 м на 11:845 Исходные размеры м.

Длина прямоугольника на 8 см больше его ширины. Если его длину и ширину увеличить на 2 см, его площадь увеличится на 68 см 2 . Найдите его первоначальные размеры.

Новая площадь на 68 больше старой.
(ш + 10)(ш + 2) = ш 2 + 8ш + 68
ш 2 + 2w + 10w + 20 = w 2 + 8w + 68
12w + 20 = 8w + 68    Добавить — 8w
4w + 20 = 68
4w = 48
w = 12 : 903:4 Исходные размеры см на 20 см.

Плотник строит две колоды одинаковой площади. Одна палуба представляет собой квадрат, а другая представляет собой прямоугольник, который на 4 м длиннее квадрата, но на 3 м меньше его ширины. Какова площадь каждой колоды?

 

Длина

Width

Area

Square

n

n

n 2

Rectangle

n + 4

n — 3

n 2 + n — 12

Так как площади равны, то получаем
n 2 = n 2 + n — 12
0 = n — 12
12 = n
Ответ:
Площадь квадратного настила = 12 2 = 144 м 2
Площадь прямоугольного настила = (12 + 4)(12 — 3) = (16)(9) = 144 м 2

[Дом]

Математическая задача: Прямоугольники — стороны

Одна сторона прямоугольника на 10 см длиннее второй. Если укоротить длинную сторону на 6 см и удлинить короткую на 14 см, площадь прямоугольника увеличится на 130 см 2 .

Каковы размеры исходного прямоугольника?

Правильный ответ:

a = 19,25 см
b = 9,25 см

Пошаговое объяснение:

a=10+b (a−6)(b+14)=ab+130 ab− 6b+14a−6⋅14=ab+130 14a−6b=214 14a−6(a−10)=214 8a=154 a=19,25 см

b=a−10=9,25 см


Вы нашли ошибка или неточность? Не стесняйтесь

пишите нам

. Благодарю вас!

Советы по связанным онлайн-калькуляторам

У вас есть система уравнений и вы ищете калькулятор системы линейных уравнений?

Вам необходимо знать следующие знания для решения этой задачи по математике:

  • Алгебра
  • SYSTER
  • PLANIMETRICS
  • PLANIMETRICS
  • . задача:
    • практика для 12-летних
    • практика для 13-летних
    • практика для 14-летних

     

    Мы рекомендуем вам посмотреть это обучающее видео по этой математической задаче: видео1

    • Прямоугольная 5611
      Прямоугольная трасса на 12 м длиннее своей ширины. Предположим, что его длина увеличилась на 10 м, а площадь увеличилась на 600 квадратных метров. Каковы его размеры?
    • Прямоугольное поле
      Одно измерение прямоугольного поля на 56 м больше второго измерения. Если увеличить каждую сторону прямоугольника на 10 м, площадь поверхности составит 1480 м². Найдите размеры поля.
    • Прямоугольник
      У прямоугольника одна сторона на 8 см меньше, чем у шрифта. Если длину уменьшить на 6 см, а ширину увеличить на 2 см, то получится квадрат, площадь которого равна 400 см². Каковы первоначальные размеры прямоугольника?
    • Прямоугольник и квадрат
      Одна сторона прямоугольника на 1 см короче стороны квадрата. Вторая сторона на 3 см длиннее стороны квадрата. Квадрат и прямоугольник имеют одинаковую площадь. Вычислите длины сторон квадрата и прямоугольника.
    • Прямоугольник — стороны
      Площадь прямоугольника 266 см². Длина меньшей стороны на 5 см меньше длины большей стороны. Чему равен периметр прямоугольника?
    • Стороны прямоугольника
      Размеры прямоугольника находятся в соотношении 4:12. Чему равна длина большей стороны в сантиметрах, если меньшая сторона равна 12 см?
    • Зал
      Зал имел прямоугольную форму в плане, один размер на 20 м длиннее другого. После перестройки длина зала уменьшилась на 5 м, а ширина увеличилась на 10 м. Площадь этажа увеличилась на 300 м². Каковы были первоначальные размеры зала?
    • Два тела
      Прямоугольник с размерами 8 см и 4 см сначала поворачивают на 360º вокруг более длинной стороны, чтобы сформировать первое тело. Затем мы аналогичным образом поворачиваем прямоугольник вокруг более короткой стороны b, чтобы сформировать второе тело. Найдите отношение поверхностей первого и второго бо
    • Параллелограмм
      Периметр параллелограмма 417 см. Длина одной стороны в 1,7 раза больше длины меньшей стороны. Какова длина сторон параллелограмма?
    • Четырехугольник 45081
      Каков периметр четырехугольника, у которого первая сторона на 3 см длиннее второй, третья на 5 см короче четвертой, а вторая на 2 см короче четвертой?
    • Прямоугольники
      Периметр прямоугольника равен 90 м. Разделите его на три прямоугольника. У более короткой стороны все три прямоугольника одинаковы. Их более длинные стороны представляют собой три последовательных натуральных числа. Каковы размеры каждого прямоугольника?
    • Размеры 3159
      Прямоугольник на 6 см длиннее ширины. Квадрат со стороной, равной длине прямоугольника, имеет площадь на 78 см² больше, чем прямоугольник. Вычислите размеры прямоугольника.
    • Вариации 26791
      Если количество элементов увеличивается на два, увеличивается количество вариаций второго класса этих элементов, созданных на 38. Каково исходное количество элементов?
    • Окружность 6525
      Окружность треугольника 104 см. Одна сторона на 6 см длиннее и на 8 см короче третьей. Укажите длину страницы.
    • Окружность 5254
      Вычислите меньшую сторону и диагональ прямоугольника, если одна сторона на 2 см длиннее другой, а длина окружности равна 70 см.
    • Прямоугольник
      Длина одной стороны прямоугольника в три раза больше длины второй стороны. Каковы размеры прямоугольника, если длина его окружности равна 9?6 см?
    • Параллелограмм
      Ромбовид (параллелограмм) имеет более длинную сторону длиной 50 см. Размер его одной высоты в четыре раза больше размера его второй высоты. Вычислите длину меньшей стороны этого ромба в сантиметрах.

    геометрия — Почему площадь квадрата не всегда больше длины одной из его сторон?

    Спросил

    Изменено 9 месяцев назад

    92 < s$, что не имеет для меня смысла по указанной выше причине.

    • геометрия

    $\endgroup$

    13

    $\begingroup$

    Я думаю, ваша интуиция подводит вас, потому что вы пытаетесь сравнить одномерный объект (длину стороны) с двухмерным объектом (площадь внутренней части квадрата). Вы можете поместить нагрузок сегментов в квадрат любого размера — на самом деле бесконечно много! Это сравнение на самом деле ничего не значит.

    С другой стороны, вот сравнение, которое имеет смысл: расположите квадрат со стороной $s$ рядом с прямоугольником, сторона которого равна $s \times 1$. Теперь вы сравниваете район с районом. Площадь прямоугольника поместится внутри квадрата тогда и только тогда, когда $s>1$.

    $\endgroup$

    5

    $\begingroup$

    Потому что вы неправильно понимаете юниты. Первое предположение, которое вы делаете, состоит в том, что квадрат со стороной $1$ имеет площадь $1$ — это предположение неверно. 92$ (квадратных мегаметров), в зависимости от того, как вы выбрали для представления . Все дело в представлении, а не в математических свойствах.

    Что вам нужно интуитивно понять, так это то, что удвоив длину стороны квадрата, вы получите в 4 раза больше площади. А уменьшая сторону наполовину, вы уменьшаете площадь до четверти, независимо от единиц измерения.

    Когда у вас появится это интуитивное понимание, оно превзойдет ваше текущее понимание. Зная, что площади сжимаются «быстрее», чем длины сторон, будет очевидно, что на квадрате со стороной $1$ грок и площадь 1$ грикк , когда вы уменьшаете длину стороны, площадь должна уменьшаться быстрее, чем длина стороны — то же самое верно для квадрата с длиной стороны 42$ грикк и площадью $42$ grakk : площадь будет уменьшаться быстрее, чем длина стороны. 2$. Это имеет смысл. Я покажу вам, что я имею в виду. 92$.

    Вы всегда можете взять квадрат со стороной $x$, равной $00$. Таким образом, площадь теперь имеет смысл.

    $\endgroup$

    1

    $\begingroup$

    Вы сравниваете яблоки с апельсинами. Длины и площади измеряются в разных единицах.

    Самый интуитивный способ — рассматривать единицы как часть измерения. Таким образом, длина может быть три метров (то же самое, что 118,11 дюймов ), но не только три. Площадь = длина в квадрате, что девять квадратных метров (а не просто девять). Теперь вы можете видеть, что 118,11 намного больше, чем 9, , если единицы измерения не включены как часть измерения .

    Чтобы иметь возможность их сравнить, возьмите площадь обоих. Например, какова площадь одной из сторон квадрата? Поскольку стороны являются линиями, их толщина равна нулю, и, следовательно, они не имеют площади (или нулевой площади). 92$. И геометрически говоря:

    • Вы можете поместить любой сегмент в соответствующий ему квадрат, «скопировав» его вдоль перпендикулярной оси.
    • Вы также можете вписать любой сегмент в другой квадрат, даже если диагональ меньше сегмента. Поскольку отрезок бесконечно тонкий, его можно сложить внутрь квадрата.
    • Что еще удивительно, можно развернуть любой квадрат (или куб, или гиперкуб…) и он все равно влезет в любой отрезок. См. кривые заполнения пространства.

      $, $\mathbb{R}$ и $\mathopen{[} 0, 1 \mathclose{]}$ имеют одинаковую мощность, поэтому можно считать, что квадрат и его сторона имеют одинаковое количество точек.

      $\endgroup$

      3

      $\begingroup$

      Сначала физика, потом математика. 2>a.$

      $\endgroup$

      1

      Твой ответ

      Зарегистрируйтесь или войдите в систему

      Зарегистрируйтесь с помощью Google

      Зарегистрироваться через Facebook

      Зарегистрируйтесь, используя электронную почту и пароль

      Опубликовать как гость

      Электронная почта

      Требуется, но не отображается

      Опубликовать как гость

      Электронная почта

      Требуется, но не отображается

      Нажимая «Опубликовать свой ответ», вы соглашаетесь с нашими условиями обслуживания, политикой конфиденциальности и политикой использования файлов cookie

      .

      Площадь прямоугольника | Вычисление длины, ширины и примеры

      Прямоугольник — это тип четырехугольника, который похож на квадрат тем, что у него равные противоположные стороны и четыре прямых угла. Но, в отличие от квадрата, у которого длина и ширина одинаковы, у прямоугольника разные размеры длины и ширины.

      Здесь у нас есть прямоугольник, покрытый маленькими квадратиками. Каждый маленький квадрат соответствует 1 квадратному сантиметру, то есть каждый маленький квадрат равен сантиметрам с каждой стороны.

      Подсчитайте количество квадратов, покрывающих каждый прямоугольник. Это даст вам площадь прямоугольника в квадратных сантиметрах. Также напишите длину и ширину каждого прямоугольника. Что вы наблюдаете? Давай выясним.

      прямоугольник 1

      Количество небольших квадратов = 18

      Зона = 18 см 2 или 18 кв. См

      Длина = 6 см

      . маленькие квадраты = 20

      Площадь = 20 см 2 или 20 кв. см

      Длина = 5 см

      Ширина = 4 см

      Из приведенных выше примеров видно, что есть более быстрый способ найти площадь в каждом случае без подсчета количества квадратов. Поскольку 6 х 3 = 18 и 5 х 4 = 50, мы можем сказать, что площадь каждого из двух прямоугольников можно узнать, перемножив меры длины и ширины прямоугольника.

      Площадь прямоугольника с длиной ‘ l ‘ и шириной ‘b’ равна l x b

      Например, рассмотрим прямоугольник длиной 8 см и шириной 7 см, как показано на рисунок ниже.

      Площадь этого прямоугольника равна 8 7 7 = 56 см 2 .

      Отсюда

      Площадь прямоугольника = длина x ширина

      Теперь мы знаем, как вычислить площадь, если у нас есть длина и ширина прямоугольника.   Но если известны только длина или ширина и площадь, то можно ли узнать ширину прямоугольника? Давайте узнаем.

      С тех пор, площадь прямоугольника = длина x ширина,

      Следовательно,

      Длина = область / ширина, ширина = область / длина

      Пример 1

      Найдите высоту из высоты

      .

      стена длиной 4 м, которую можно покрыть 2400 плитками размером 25 на 20 см.

      Solution              

      Area of ​​tiles = (25 x 20) cm 2 = 500 cm 2

      Therefore, area of ​​2400 tiles = 2400 x 500 cm 2

                                                  = 1200000 cm 2

      = 1200000 /10000 м 2 (10000 см 2 = 1 м 2 )

      = 120 М 2

      Пусть высота стены будет «h» метров. Тогда

      Площадь стены = 4 x h м 2 = 4h м 2

      Так как 2400 плиток полностью покрывают стену, следовательно,

      Площадь стены = Площадь 2400 плиток

      ⇒ 04h

      ⇒ h =  = 30

      Отсюда высота стены 30 метров.

      Пример 2

          

      Найдите площадь в квадратных метрах прямоугольника, длина которого

      а) равна 4,5 м, а ширина = 1,6 м.

      b) длина = 3 дм 6 см, ширина = 2 дм 9 см

      Решение              

      Отсюда, Площадь прямоугольника = (4,5 х 1,6) м 2 = 7,20 м 2

      б) Имеем,

      Длина = 3 дм 6 см + = (3) см = 36 см  = 0,36 м (∵ 1 дм = 10 см)

      Ширина = 2 дм 9 см = (2 x 10 + 9CM = 29 см = 0,29 м

      Площадь прямоугольника = длина x ширина

      Следовательно, площадь прямоугольника = (0,36 x 0,29) M 2 = 0,1044 M 2

      Пример 3

      9 2

      Пример 3

      2

      .

      Травяной участок размером 80 м х 60 м. Две пересекающиеся дорожки шириной по 4 м построены под прямым углом через центр поля так, что каждая дорожка параллельна одной из сторон прямоугольников. Найдите общую площадь используемого пути. Кроме того, найдите стоимость посыпки их гравием в размере 5 фунтов стерлингов за квадратный метр.

      Решение              

      Важной стратегией решения таких вопросов является их визуализация. Поэтому мы построим прямоугольник и построим детали, которые нам дали в вопросе.

      Пусть ABDC и PQSR — прямоугольные пути, которые должны быть построены под прямым углом через центр поля так, чтобы каждый путь был параллелен одной из сторон прямоугольников. У нас есть,

      Нам нужно найти общую площадь используемого пути.

      Сначала вычислим площадь пути ABDC.

      Так как травяной участок представляет собой прямоугольник, и мы знаем, что площадь прямоугольника, имеющего длину ‘ l ‘ и ширину ‘b’, равна l x b, следовательно,

      Площадь пути ABDC =(80 x 4) м 2 = 320 м 2

      Теперь давайте снова найдем площадь пути PQSR, которую можно рассчитать так же, как это было сделано выше.

      Имеем,

      Площадь пути PQSR = (60 х 4) м 2 = 240 м 2

      Но с помощью нашего рисунка мы можем ясно видеть, что площадь EFHG является общей для обеих площадей, которые мы только что вычислили. Следовательно, нам нужно вычесть эту площадь один раз из нашего результата, чтобы не считать ее дважды.

      Площадь пути EFHG = (4 x 4) м 2 = 16 м 2

      Следовательно,

      Общая площадь пути = Площадь пути ABDC + Площадь пути PQSR – Площадь путь EFHG

      Следовательно,

      Общая площадь дорожки = (320 + 240 – 16 ) м 2 = 544 м 2

      Теперь посчитаем стоимость засыпки дорожки гравием.

      Нам сообщили, что стоимость покрытия дорожки гравием составляет 5 фунтов стерлингов за квадратный метр.

      Так как общая площадь, которую нужно посыпать гравием, составляет 544 м 2 , следовательно,

      Общая стоимость посыпки дорожки гравием составит фунтов стерлингов (544 x 5) = 2720 фунтов стерлингов.

      Пример 4

          

      Прямоугольная дорожка имеет длину 45 м и ширину 30 м. За пределами парка проложена дорожка шириной 2,5 м. Найдите площадь пути.

      Решение              

      Пусть ABCD — прямоугольная дорожка, а заштрихованная область представляет собой дорожку шириной 2,5 м, которую нужно построить за пределами парка.

      Имеем,

      PQ = (45 +2,5 + 2,5) м = 50 м

      PS = (30 + 2,5 + 2,5) м = 35 м

      Площадь прямоугольника PQRS = (50 x 35) м 2 = 1750 м 2

      Площадь прямоугольника ABCD = (45 x 30) M 2 = 1350 M 2

      Очевидно,

      Площадь пути = Площадь прямоугольника PQRS — Площадь прямоугольника ABCD

      = 1750 М. 2 — 1350 M 2

      = 400 м 2

      Следовательно, площадь пути = 400 М 2

      Пример 5

      9. площадь = 90см 2 и ширина = 6 см

      Решение

      Нам дается, что площадь = 90 см 2 и ширина = 10 см

      Сейчас, Длина = область/Флотная 6  см = 15 см

      Следовательно, длина прямоугольника с площадью = 90 см 2 и шириной = 6 см равна 15 см

      Комната длиной 5,5 м и шириной 4 м.

      Найти

      а) Площадь галереи

      б) Стоимость цементирования пола галереи из расчета 20 фунтов стерлингов за м информацию через графическое представление. Пусть ABCD представляет собой пол комнаты, а заштрихованная область представляет собой пол галереи шириной 1,25 м.

      Имеем

      PQ = (5,5 + 1,25 + 1,25) m = 8 м и

      PS = (4 + 1,25 + 1,25) m = 6,5 м

      Теперь,

      Площадь прямоугольника PQRS = PQ X PS

      = 8 x 6,5 м 2

      = 52 M 2

      Площадь прямоугольника ABCD = AB X

      = (5,5 x x. 4) м 2

      = 22 м 2

      Теперь

      Площадь галереи = Площадь прямоугольника PQRS – Площадь прямоугольника ABCD

      = (52 – 22) м 2 = 30 м 2

      Отсюда площадь галереи = 30 м 2

      б) Теперь, когда мы вычислили площадь галереи необходимо узнать стоимость цементирования галереи.

      Нам известно, что

      Стоимость цементирования пола галереи = 20 фунтов стерлингов за м 2  

      Следовательно,

      Стоимость цементирования пола галереи = £(20 x 30) = 600 фунтов стерлингов

      Следовательно, стоимость цементирования пола галереи = 600 фунтов стерлингов

      Пример 7    

      Комната имеет длину 7 м и ширину 5,5 м. Он имеет одну дверь длиной 2 м и шириной 1,5 м и два окна длиной 1,5 м и шириной 1 м каждое. Найти

      а) Площадь двери

      б) Площадь окон

      в) Площадь четырех стен

      г) Стоимость побелки стен из расчета 50 фунтов стерлингов за квадратный метр, если стены 3,5 м высокая.

      Решение              

      а) Давайте сначала разберемся, какая информация была предоставлена ​​в вопросе. Нам дано, что

      Длина двери = 2 м

      Ширина двери = 1,5 м

      Площадь двери = (2 x 1,5) м 2 = 3 м 2

      Следовательно, площадь двери = 3 м 2

      Аналогично можно рассчитать площадь окон.

      б) Нам дано, что

      Длина одного окна = 1,5 м

      Ширина одного окна = 1 м

      Площадь одного окна = (1,5 x 1) м 2 = 1,5 м 2

      Площадь двух окон = 1,5 x 2 = 3 м 7 2

      0

      Следовательно, площадь двух окон = 3 м 2

      c) Теперь рассчитаем площадь четырех стен. Нам дано, что

      Длина комнаты = 7 м

      Ширина комнаты = 5,5 м

      Высота комнаты = 3,5 м

      Площадь четырех стен = (Длина x Высота) + (Длина x Высота) ) + (Ширина x Высота) + (Ширина x Высота)

      Следовательно,

       Площадь четырех стен = 2 (Длина x Высота) + 2 (Ширина x Высота)

      Используя распределительное свойство над сложением, мы имеем

      Площадь четырех стен = 2 ((Длина + Ширина) х Высота

      = [2 (7 + 5,5) х 3,5] м 2

      = [2 х 1,5 х 3,5] м 2

      = 87,5 м 2

      стены = 87,5 м 2

      г) Теперь о стоимости побелки стен. Нам сообщили, что стоимость побелки стен составляет 50 фунтов стерлингов за квадратный метр.

      SO,

      Стоимость побели 1 м 2 = 50

      Стоимость побели 87,5 м 2 = £ (50 x 81,5) = 4075

      . Следовательно, стоимость белой массы = 5

      . Следовательно, стоимость белой массы = 5

      .

      Пример 8

          

      Прямоугольный сад и квадратный сад имеют одинаковый периметр 100 м. Если прямоугольный сад имеет ширину на 2 м меньше площади квадратного сада, найдите

      а) ширину прямоугольного сада

      б) длина прямоугольного сада

      в) площадь прямоугольного сада

      Решение              

      Нам известно, что прямоугольный сад и квадратный сад имеют одинаковый периметр. Мы знаем, что все стороны квадрата равны, а периметр квадрата = 4 x Сторона.

      Следовательно, мы можем сказать, что

      4 x Сторона = 100 м

      Сторона = 100/4 = 25 м

      Следовательно, сторона квадрата = 25 м.

      Нам также известно, что прямоугольный сад имеет ширину на 2 см меньше, чем квадратный сад. Это означает, что ширина прямоугольного сада = 25 – 2 = 23 м.

      Следовательно, ширина прямоугольного сада = 23 м

      б) Теперь нам нужно найти длину прямоугольного сада. Теперь мы знаем ширину прямоугольного сада и его периметр, так как прямоугольный сад и квадратный сад имеют одинаковый периметр. Мы также знаем, что

      Периметр прямоугольника = 2 (длина + ширина)

      Следовательно,

      2 (длина + 23) = 100 м

      2 x длина + 46 = 100 м

      2 x Длина = 54 м

      Длина = 54/2 м = 27 м

      Отсюда длина прямоугольного сада = 27 м

      в) Далее нужно найти площадь прямоугольного участка сад. Из предыдущих расчетов имеем и длину, и ширину прямоугольного сада, т.е.

      Длина прямоугольного сада = 27 м

      Ширина прямоугольного сада = 23 м

      Мы знаем, что

      Площадь прямоугольника = Длина x Ширина

      Следовательно,

      Площадь прямоугольного сада = (27 x 23) M 2

      = 621 M 2

      Площадь прямоугольного сада = 621 M 2

      Пример

      Журнал берет за рекламу 300 фунтов стерлингов за 10 кв. см площади. Компания решила заказать рекламу в полполосы. Если каждая страница журнала имеет размер 15 см х 24 см, какую сумму компания заплатит за рекламу?

      Решение              

      Нам известно, что размеры каждой страницы журнала составляют 15 см x 24 см. Это означает, что журнал имеет прямоугольную форму. Примем большую сторону магазина за длину, а короткую за ширину. У нас есть,

      Длина одной страницы журнала = 24 см

      Ширина одной страницы журнала = 15 см

      Мы знаем, что

      Площадь прямоугольника = длина x ширина

      Следовательно,

      Площадь одной страницы журнала = (24 x 15) см 2 = 360 см 2

      Площадь полстраницы журнала = 1/2 x 360 см 2 = 180 см 2

      10 кв. см рекламы = 300 фунтов стерлингов

      Следовательно, плата за 1 кв. см рекламы = 300 фунтов стерлингов/100 = 30 фунтов стерлингов

      Теперь,

      Плата за полстраницы рекламы  = Площадь половины страницы журнала x Плата за 10 кв. см рекламы

      Следовательно,

      Плата за половину страницы рекламы  = £ (180 x 30) = £ 5400

      Таким образом, компания должна заплатить £5400 за рекламу на полстраницы

      Пример 10

      Укажите Верно или Неверно следующее утверждение и аргументируйте свой ответ.

      Утверждение – «Если длину прямоугольника разделить пополам, а ширину удвоить, то площадь полученного прямоугольника останется прежней»

      Решение               

      Пусть имеется прямоугольник длины «l» и ширины «b». Теперь мы знаем, что

       Площадь прямоугольника = Длина x Ширина = l x b ………………………………………….. (1)

      Теперь нам известно, что длина нового прямоугольник разделен пополам.

      Таким образом, длина прямоугольника теперь равна l/2

      Кроме того, ширина прямоугольника удваивается. Это означает, что ширина прямоугольника теперь становится равной 2b.

      Площадь прямоугольника теперь будет = l/2 x 2b = l x b………………………………………. .(2)

      (1) и (2) совпадают, это означает, что утверждение «Если длину прямоугольника уменьшить вдвое, а ширину — вдвое, то площадь полученного прямоугольника останется прежней» верно.

      Площадь квадратов и прямоугольников (на тему недвижимости) Рабочие листы
      Решение текстовых задач на периметр и площадь прямоугольника Рабочие листы по математике 4 класса
      Формы разделов (прямоугольники и круги) Рабочие листы по математике для 2-го класса

      Просмотреть все рабочие листы

      Мы тратим много времени на изучение и сбор информации на этом сайте. Если вы сочтете это полезным в своем исследовании, используйте приведенный ниже инструмент, чтобы правильно указать ссылку Helping with Math в качестве источника. Мы ценим вашу поддержку!

      Решения NCERT для класса 7 по математике, глава 11

      • Решения НЦЭРТ
      • Класс 7
      • Математика
      • периметр и площадь

      Математика NCERT 7 класс, Глава 11: Периметр и площадь — В этой главе приведены все подробности о периметре и площади плоских фигур . Сначала в главе обсуждаются квадратов и прямоугольников.

      • Периметр — это расстояние вокруг замкнутой фигуры , тогда как площадь — это часть плоскости, занимаемая замкнутой фигурой.

      Периметр следующих геометрических фигур обсуждается в первом разделе:

      • Квадрат
        • Периметр квадрата = 4 × сторона
      • Прямоугольник
        • Периметр прямоугольника = 2 × (длина + ширина)

      Раздел разделен на две части:
      1. Треугольники как части прямоугольников
      2. Обобщение для других конгруэнтных частей прямоугольников
      Прежде чем перейти к упражнению 11.2, обсуждаются темы Площадь треугольника и параллелограмма .

      • Площадь квадрата = сторона × сторона
      • Площадь прямоугольника = длина × ширина
      • Площадь параллелограмма = основание × высота
      • Площадь треугольника =1/2 × основание × высота

      Во второй половине главы объясняются кругов . Тема объясняется в двух частях:
      1 . Окружность круга

      • Расстояние вокруг круговой области известно как ее окружность.
      • Длина окружности = π d

      2. Площадь круга
      Площадь круга определяется как π r 2

      Последняя часть главы посвящена преобразованию единиц и применению формул.

      • На основе преобразования единиц длины , изученного в главе, единиц площади также могут быть преобразованы:
      • 1 см 2 = 100 мм 2  
      • 1 м 2 = 10000 см 2
      • 1 га = 10000 м 2

      Различные формулы и преобразования единиц пересматриваются, и поэтому в этой теме изучается их применение в сценариях реальной жизни. Эти вопросы требуют практики, и поэтому нужно попытаться решить как можно больше вопросов в этой главе.
      В конце учащиеся будут проинформированы о важных моментах главы периметра и области .

       

      Страница № 208:
      Вопрос 1:

      Длина и ширина прямоугольного участка земли равны 500 м и 300 м соответственно. Найдите

      (i) его площадь

      (ii) стоимость земли, если 1 м 2 земли стоит 10 000 рупий.

      Ответ:

      (i) Площадь = длина × ширина

      = 500 × 300

      = 150000 м 2

      (ii) Стоимость 1 м 2 Земля = 10000

      Стоимость 150000 М 2 Земля = 10000 × 150000 = RS 1500000000

      Страница № 2089999999999999999
    • 0899.08908999999999999999999999.9999999999999999999999.9999999999999999999900 года 90890890899089
      8908

      0

      9
      . :

      Найти площадь квадратного парка периметром 320 м.

      Ответ:

      Периметр = 320 м

      4 × Длина стороны парка = 320

      Длина сторона парка =

      Площадь = (Длина стороны парка) 2 = (80) 2 = 6400 м 2

      Номер страницы 208:
      Вопрос 3:

      Найдите ширина прямоугольного земельного участка, если его площадь 440 м 2 и длина 22 м. Также найдите его периметр.

      Ответ:

      Площадь = Длина × Ширина = 440 м 2

      22 × Ширина = 440

      Ширина == 20 м

      Периметр = 2 (длина + ширина)

      = 2 (22 + 20) = 2 (42) = 84 м 2

      Страница № 208:
      Вопрос 4:

      периметр прямоугольного листа равен 100 см. Если длина 35 см, найти его широту. Также найдите площадь.

      Ответ:

      Периметр = 2 (Длина + Ширина) = 100 см

      2 (35 + Ширина) = 100

      35 + B = 50

      В = 50 — 35 = 15 см

      Площадь = Длина × Ширина = 35 × 15 = 525 см 2

      Номер страницы 208:
      Вопрос 5:

      Площадь квадратного парка такая же, как и прямоугольного. Если сторона квадратного парка 60 м, а длина прямоугольного парка 90 м, найдите ширину прямоугольного парка.

      Ответ:

      Площадь квадратного парка = (Одна из его сторон) 2 = (60) 2 = 3600 м 2

      Площадь прямоугольного парка = Длина × Ширина = 3600

      5 90 = 3600

      Ширина = 40 м
      ​​​​​​​​

      Видео Решение для периметра и площади (Страница: 208 , Q.
      No.: 5)

      NCERT Решение для математики класса 7 — периметр и район 208 , Вопрос 5

      Страница № 208:
      Вопрос 6:

      Провод имеет форму прямоугольника. Его длина 40 см, а ширина 22 см. Если ту же проволоку согнуть в форме квадрата, какова будет мера каждой стороны? Также найдите, какая фигура охватывает большую площадь?

      Ответ:

      Периметр прямоугольника = Периметр квадрата

      2 (Длина + Ширина) = 4 × Сторона

      2 (40 + 22) = 4 × Сторона

      2 × 62 = 4 × Сторона = 31 см

      Площадь прямоугольника = 40 × 22 = 880 см 2

      Площадь квадрата = (Сторона) 2 = 31 × 31 = 961 см 2

      Следовательно, провод квадратной формы занимает больше площади.
      ​​​​​​

      Видеорешение для периметра и площади (Страница: 208 , Q.No.: 6)

      Решение NCERT для математики класса 7 — периметр и площадь 208 , Вопрос 6

      Страница № 208 :
      Вопрос 7:

      периметр прямоугольника 130 см. Если ширина прямоугольника 30 см, найдите его длину. Также найдите площадь прямоугольника.

      Ответ:

      Периметр = 2 (длина + ширина) = 130

      2 (длина + 30) = 130

      Длина + 30 = 65

      Длина = 65 − 30 = 35 см

      Площадь = Длина × Ширина = 35 × 30 = 1050 см 2

      Номер страницы 208:
      Вопрос 8:

      Дверь из длиной 2 м и шириной 1 м вмонтирован в стену. Длина стена 4,5 м, а ширина 3,6 м (см. рисунок). Находить стоимость побелки стены, если тариф на побелку стены стены 20 рупий за м 2 .

      Ответ:

      Площадь стена = 4,5 × 3,6 = 16,2 м 2

      Площадь дверь = 2 × 1 = 2 м 2

      побеленный = 16,2 − 2 = 14,2 м 2

      Стоимость побелка 1 м 2 площадь = 20

      ∴Стоимость побелки 14,2 м 2 площадь = 14,2 × 20 = 284

      рупий Страница № 216:
      Вопрос 1:

      Найти площадь каждого из следующих параллелограммов:

      Ответ:

      Площадь параллелограмм = основание × высота

      (a) Высота = 4 см

      Основание = 7 см

      Площадь параллелограмма = 7 × 4 = 28 см 2

      (b) Высота = 3 см

      Основание = 5 см

      Площадь параллелограмма = 5 × 3 = 15 см 2

      (c) Высота = 3,5 см

      Основание = 2,5 см

      Площадь параллелограмма = 2,5 × 3,5 = 8,75 см 2

      (d) Высота = 4,8 см

      Основание = 5 см

      Площадь параллелограмма = 5 × 4,8 = 24 см 2

      (e) Высота = 4,4 см

      Основание = 2 см

      Площадь параллелограмма = 2 × 4,4 = 8,8 см 2

      Номер страницы 216:
      Вопрос 2:

      площадь каждого из следующих треугольников:

      Ответ:

      (a) Основание = 4 см, высота = 3 см

      Область = 6 см 2

      (b) Основание = 5 см, высота= 3,2 см

      Область = 8 см 2

      (c) Основание = 4 см, высота = 3 см

      Область = 6 см 2

      (d) Основание = 3 см, высота = 2 см

      Площадь = = 3 см 2

      Номер страницы 216:
      Вопрос 3:

      Найдите недостающие значения:

      Итак №

      Основание

      Высота

      Площадь параллелограмма

      а.

      20 см

      246 см 2

      б.

      15 см

      154,5 см 2

      с.

      8,4 см

      48,72 см 2

      д.

      15,6 см

      16,38 см 2

      Ответ:

      Площадь параллелограмма = Основание × Высота

      (a) b = 20 см

      h = ?

      Площадь = 246 см 2

      20 × ч = 246

      Следовательно, высота такого параллелограмма 12,3 см.

      (б) б = ?

      H = 15 см

      Площадь = 154,5 см 2

      B × 15 = 154,5

      B = 10,3 CM

      Следовательно, базовая база.

      (в) б = ?

      ч = 8,4 см

      Площадь = 48,72 см 2

      b × 8,4 = 48,72

      Следовательно, основание такого параллелограмма равно 5,8 см.

      (г) б = 15,6 см

      ч = ?

      Площадь = 16,38 см 2

      15,6 × h = 16,38

      Следовательно, высота такого параллелограмма 1,05 см.

      Страница № 217:
      Вопрос 4:

      Найдите недостающие значения:

      Основание

      Высота

      Площадь треугольника

      15 см

      _______

      87 см 2

      _______

      31,4 мм

      1256 мм 2

      22 см

      _______

      170,5 см 2

      Ответ:

      (a) b = 15 см

      h = ?

      Площадь =

      Следовательно, высота такого треугольника 11,6см.

      (б) б = ?

      h = 31,4 мм

      Площадь =

      Следовательно, основание такого треугольника равно 80 мм.

      (в) б = 22 см

      ч = ?

      Площадь =

      Следовательно, высота такого треугольника 15,5см.

      Страница № 217:
      Вопрос 5:

      PQRS является параллелограммом (см. рисунок). QM — высота от Q до SR, а QN — высота от Q до PS. Если SR = 12 см и QM = 7,6 см. Найти:

      (а) площадь параллелограмма PQRS

      (б) QN, если PS = 8 см

      Ответ:

      (a) Площадь параллелограмма = Основание × Высота = SR × QM

      = 7,6 × 12 = 91,2 см 2

      (b) Площадь параллелограмма = Основание × Высота = PS × QN = 91,2 см 2

      QN × 8 = 91,2


      ​​​​​​

      Видео Решение для периметра и площади (Страница: 217 , Q.№: 5)

      NCERT Решение для Математика 7 класс — периметр и площадь 217 , Вопрос 5

      Номер страницы 217:
      Вопрос 6:

      ДЛ и БМ — высоты сторон AB и AD параллелограмма ABCD соответственно. (см. приведенный рисунок). Если площадь параллелограмма 1470 см 2 , АВ = 35 см и AD = 49 см, найдите длину BM и DL.

      Ответ:

      Площадь параллелограмм = основание × высота = AB × DL

      1470 = 35 × DL

      Также 1470 = АД × БМ

      1470 = 49 × BM

      Страница № 217:
      Вопрос 7:

      ΔABC образует прямой угол в точке A (см. рисунок). AD перпендикулярна BC. Если АВ = 5 см, ВС = 13 см и АС = 12 см, найдите площадь ΔАВС. Также найдите длину AD.

      Ответ:

      Площадь == 30 см 2


      ​​​​​​​​​

      Видео Решение для периметра и площади (Страница: 217 , Q.No.: 7 )

      NCERT Решение по математике для 7 класса — периметр и площадь 217 , Вопрос 7

      Страница № 217:
      Вопрос 8:

      ΔАВС равнобедренный с AB = AC = 7,5 см и BC = 9 см (см. фигура). Высота AD от А до ВС равна 6 см. Найдите площадь ΔАВС. Какова будет высота от C до AB, т. е. CE?

      Ответ:

      Номер страницы 223:
      Вопрос 1:

      Найти окружность кругов следующего радиуса: (Возьмем π)

      (а) 14 см (б) 28 мм (в) 21 см

      Ответ:

      (а) r = 14 см

      Окружность = 2π r = 88 см

      (б) р = 28 мм

      Окружность = 2π r = 176 мм

      (в) r = 21 см

      Окружность = 2π r = 132 см

      Страница № 223:
      Вопрос 2:

      Найдите площадь следующих кругов, учитывая, что:

      (a) радиус = 14 мм (Примите π) (б) диаметр = 49 м

      (в) радиус = 5 см

      Ответ:

      (а) р = 14 мм

      Площадь = π r 2 = 616 мм 2

      (б) d = 49 м

      г =

      Площадь = π r 2 == 1886,5 м 2

      (в) r = 5 см

      Площадь = π r 2 = = 78,57 см 2

      Страница № 223:
      Вопрос 3:

      Если длина окружности круглого листа равна 154 м, найдите его радиус. Также найти площадь

      лист. (Возьмите π)

      Ответ:

      Окружность = 2π r =154 м

      Площадь = π r 2 знак равно

      знак равно = 1886,5 м 2

      Страница № 223:
      Вопрос 4:

      Садовник хочет огородить круглый сад диаметром 21 м. Найдите длину веревку ему нужно купить, если он сделает 2 круга забора. Также Найдите стоимость веревки, если она стоит 4 рубля за метр. (Возьмите π)

      Ответ:

      д = 21 м

      р =

      Окружность = 2π r == 66 м

      Длина для ограждения требуется веревка = 2 × 66 м = 132 м

      Стоимость 1 шт. м веревки = 4

      рупий Стоимость 132 м веревки = 4 × 132 = 528 рупий

      Страница № 223:
      Вопрос 5:

      Из круговой лист радиусом 4 см, круг радиусом 3 см удаляется. Найдите площадь оставшегося листа. (Возьмите π = 3,14)

      Ответ:

      Внешний радиус круглого листа = 4 см

      Внутренний радиус круглого листа = 3 см

      Остаток площадь = 3,14 × 4 × 4 — 3,14 × 3 × 3

      = 50,24 — 28,26

      = 21,98 см 2

      Страница № 223:
      Вопрос 6:

      Saima хочет положить шнурок на край круглой крышки стола диаметром 1,5 м. Найдите необходимую длину шнурка, а также его стоимость, если один метр кружева стоит 15 рупий. = 3,14)

      Ответ:

      Окружность = 2π r

      Стоимость 1 шт. м кружева = 15 рупий

      Стоимость 4,71 м кружева = 4,71 × 15 = 70,65 рупий

      Номер страницы 223:
      Вопрос 7:

      Найдите периметр соседней фигуры, представляющей собой полукруг, включающий его диаметр.

      Ответ:

      Радиус = 5 см

      Длина изогнутая часть = π r

      = 15,71 см

      Итого периметр = длина изогнутой части + длина диаметра

      = 15,71 + 10 = 25,71 см

      Страница № 223:
      Вопрос 8:

      Найдите стоимость шлифовки круглой столешницы диаметром 1,6 м, если тариф полировка 15 рупий/м 2 . (Возьмите π = 3.14)

      Ответ:

      Диаметр = 1,6 м

      Радиус знак равно 0,8 м

      Площадь = 3,14 × 0,8 × 0,8

      = 2,0096 м 2

      Стоимость за полировка 1 м 2 площадь = 15

      рупий Стоимость за полировка 2. 0096 м 2 площадь = 15 × 2,0096 = 30,14

      Следовательно, полировка такого круглого стола будет стоить 30,14 рупий.

      Страница № 223:
      Вопрос 9:

      Шазли взял проволоку длиной 44 см и согнул ее в форме круга. Найдите радиус этой окружности. Также найдите его площадь. Если тот же провод сгибается в форме квадрата, какова будет длина каждого из его сторон? Какая фигура занимает большую площадь, круг или площадь? (Возьми π)

      Ответ:

      Окружность = 2π r = 44 см

      r = 7 см

      Площадь = π r 2

      Если проволока согнута в квадрат, тогда длина каждой стороны будет =

      Площадь квадрат = (11) 2 = 121 см 2

      Следовательно, круг охватывает большую площадь.

      Страница № 223:
      Вопрос 10:

      Из круглого карточного листа радиусом 14 см удаляются два круга радиусом 3,5 см и прямоугольник длиной 3 см и шириной 1 см (как показано на следующем рисунке) . Найдите площадь оставшегося листа. (Возьми π)

      Ответ:

      Площадь более крупного круга == 616 см 2

      Площадь из 2 маленьких кругов = 2 × π R 2 = 77 CM 2

      9

      9000 40004

      9000 4000 4000 4000 4000 4000 40304 9000 400004 9000 40339. Ширина = 3 × 1 = 3 см 2

      Оставшаяся площадь листа = 616 − 77 − 3 = 536 см 2
      ​​​​​​​​​​​​

      площадь (Страница: 223 , Q.No.: 10)

      NCERT Решение по математике для 7 класса — периметр и площадь 223 , Вопрос 10

      Страница № 224:
      Вопрос 11:

      Круг из квадратного куска алюминиевого листа вырезается радиусом 2 см. стороны 6 см. Какова площадь оставшегося алюминиевого листа? (Возьмем π = 3,14)

      Ответ:

      Площадь лист квадратной формы = (Сторона) 2 = (6) 2 = 36 см 2

      Площадь круг = 3,14 × 2 × 2 = 12,56 см 2

      Остаток площадь листа = 36 − 12,56 = 23,44 см 2

      Страница № 224:
      Вопрос 12:

      длина окружности 31,4 см. Найдите радиус и площадь круг? (Возьмем π = 3,14)

      Ответ:

      Окружность = 2π r = 31,4 см

      2 × 3,14 × r = 31,4

      r = 5 см

      Площадь = 3,14 × 5 × 5 = 78,50 см 2

      Страница № 224:
      Вопрос 13:

      Круглая клумба окружена дорожкой шириной 4 м. Диаметр клумбы 66 м. Какова площадь этого пути? (π = 3,14)

      Ответ:

      Радиус клумбы = 33 м

      Радиус клумбы и дорожки вместе = 33 + 4 = 37 м

      Площадь клумбы и дорожки вместе = 3,14 × 37 × 37 = 4298,66 м 2

      Площадь клумбы = 3,14 × 33 × 33 = 3419,46 м 2

      Площадь дорожки = Площадь клумбы и дорожки вместе − Площадь клумбы

      = 4298,66 − 3419,40 м = 490,20
      ​​​​​​​​

      Видео решение для периметра и площади (Страница: 224 , Q.No.: 13)

      NCERT Решение по математике для 7 класса — периметр и площадь 224 , Вопрос 13

      Страница № 224:
      Вопрос 14:

      Циркуляр цветник имеет площадь 314 м 2 . Разбрызгиватель на центр сада может занимать территорию радиусом 12 м. Будет ли разбрызгиватель поливать весь сад? (Возьмите π = 3.14)

      Ответ:

      Площадь = π r 2 = 314 м 2

      3,14 × г 2 = 314

      р 2 = 100

      г = 10 м

      Да, спринклер будет поливать весь сад.

      Страница № 224:
      Вопрос 15:

      Найдите окружности внутреннего и внешнего кругов, показанные на соседняя фигура? (Возьмем π = 3.14)

      Ответ:

      Радиус внешний круг = 19 м

      Окружность = 2π r = 2 × 3,14 × 19 = 119,32 м

      Радиус внутренний круг = 19− 10 = 9 м

      Окружность = 2π r = 2 × 3,14 × 9 = 56,52 м

      Номер страницы 224:
      Вопрос 16:

      Сколько сколько раз должно повернуться колесо радиусом 28 см, чтобы пройти 352 м?

      (Возьми π )

      Ответ:

      r = 28 см

      Окружность = 2π r = = 176 см

      Количество оборотов =

      Следовательно, он будет вращаться 200 раз.

      Страница № 224:
      Вопрос 17:

      Минутная стрелка круглых часов имеет длину 15 см. Какое расстояние проходит кончик минутной стрелки за 1 час? (Возьмем π = 3,14)

      Ответ:

      Расстояние, пройденное по вершине минутной руки = окружность часов

      = 2π R = 2 × 3,14 × 15

      = 94,2 см

      Страница № 226:
      Вопрос 1:

      9
      Страница № 226:
      1:

      1

      99990

      4490

      90

      90

      . Сад имеет длину 90 м и ширину 75 м. Снаружи должна быть проложена дорожка шириной 5 м. и вокруг него. Найдите площадь пути. Также найдите площадь сад в гектарах.

      Ответ:

      Длина ( l ) сада = 90 м

      Ширина ( b ) сада = 75 м

      Площадь сад = l × b = 90 × 75 = 6750 м 2

      Из Рисунок, можно заметить, что новая длина и ширина сад, когда дорожка также включена, составляют 100 м и 85 м соответственно.

      Район г. сад, включая дорожку = 100 × 85 = 8500 м 2

      Площадь дорожка = площадь сада, включая дорожку − площадь сада

      = 8500 − 6750 = 1750 м 2

      1 га = 10000 м 2

      Следовательно, площадь сада в гектарах

      Страница № 226:
      Вопрос 2:

      A Ширина 3 м дорожка проходит снаружи и вокруг прямоугольного парка длиной 125 м и ширина 65 м. Найдите площадь пути.

      Ответ:

      Длина ( l ) парка = 125 м

      Ширина ( b ) парка = 65 м

      Площадь парк = l × b = 125 × 65 = 8125 м 2

      Из Рисунок, можно заметить, что новая длина и ширина длина парка, включая дорожку, составляет 131 м и 71 м соответственно.

      Район г. парк с дорожкой = 131 × 71 = 9301 м 2

      Площадь дорожка = площадь парка, включая дорожку − площадь парка

      = 9301 − 8125 = 1176 м 2

      Страница № 226:
      Вопрос 3:

      Рисунок нарисован на картоне длиной 8 см и шириной 5 см так, что с запасом по 1,5 см с каждой стороны. Найдите общую площадь маржа.

      Ответ:

      Длина ( l ) картона = 8 см

      Ширина ( b ) из картона = 5 см

      Площадь картон с полями = l × b = 8 × 5 = 40 см 2

      От Рисунок, можно заметить, что новая длина и ширина картон, если поля не включены, составляют 5 см и 2 см соответственно.

      Район г. картон без полей = 5 × 2 = 10 см 2

      Площадь поле = площадь картона, включая поле — площадь картон не

      включая поля

      = 40 − 10 = 30 см 2

      Страница № 226:
      Вопрос 4:

      Веранда шириной 2,25 м сооружена на всем протяжении снаружи помещения, которое 5,5 м в длину и 4 м в ширину. Найти:

      (i) площадь веранды

      (ii) Стоимость цементирования пола веранды из расчета 200 руб. м 2 .

      Ответ:

      (i)

      Длина ( l ) помещения = 5,5 м

      Ширина ( b ) комнаты = 4 м

      Площадь комната = л × b = 5,5 × 4 = 22 м 2

      Из Рисунок, можно заметить, что новая длина и ширина комнаты, когда есть веранда, 10 м и 8,5 м соответственно.

      Район г. комната с верандой = 10 × 8,5 = 85 м 2

      Площадь веранда = площадь комнаты, включая веранду – площадь номер

      = 85 − 22 = 63 м 2

      (ii)

      Стоимость цементирование 1 м 2 площадь пола веранды = 200 рупий

      Стоимость цементирование 63 м 2 площадь пола веранды = 200 × 63

      = 12600

      Страница № 226:
      Вопрос 5:

      По границе и внутри квадрата построена дорожка шириной 1 м сад стороны 30 m. Находить:

      (i) площадь дорожки

      (ii) стоимость посадки травы на оставшейся части дорожки сад из расчета 40 рупий за м 2 .

      Ответ:

      (i)

      Сторона ( a ) квадратного сада = 30 м (30) 2 = 900 м 2

      Из рисунка видно, что сторона квадрата сад, если дорожка не включена, составляет 28 м.

      Площадь квадратного сада без учета дорожки = (28) 2 = 784 м 2

      Площадь дорожки = Площадь квадратного сада, включая дорожку − Площадь квадрата

      сад без учета дорожки

      = 900 − 784 = 116 м 2

      (ii)

      Стоимость посадки травы на 1 м 2 площади сада = рупий 40

      Стоимость посадки травы на 784 м 2 площади сада = 784 × 40 = 31360

      рупий
      Страница № 227:
      Вопрос 6:

      Два креста дороги, каждая шириной 10 м, срезанные под прямым углом через центр прямоугольный парк длиной 700 м и шириной 300 м, параллельный его стороны. Найдите площадь дорог. Также найдите площадь парка не считая перекрестков. Ответ дайте в гектарах.

      Ответ:

      Длина ( l ) парка = 700 м

      Ширина ( b ) парка = 300 м

      Площадь парк = 700×300 = 210000 м 2

      Длина дорога PQRS = 700 м

      Длина дорога ABCD = 300 м

      Ширина каждая дорога = 10 м

      Площадь дороги = ar (PQRS) + ar (ABCD) − ar (KLMN)

      = (700 × 10) + (300 × 10) − (10 × 10)

      = 7000 + 3000 — 100

      = 10000 − 100 = 9900 м 2 = 0,99 га

      Площадь парк без дорог = 210000 − 9900 = 200100 м 2 = 20,01 га

      Страница № 227:
      Вопрос 7:

      Через прямоугольное поле длиной 90 м и шириной 60 м проложены две дороги, параллельные сторонам и пересекающие друг друга под прямым углом через центр полей. Если ширина каждой дороги 3 м, найдите

      (i) площадь, покрытую дорогами.

      (ii) стоимость строительства дорог из расчета 110 рупий за м 2 .

      Ответ:

      Длина ( l ) поля = 90 м

      Ширина ( b 9 2

      Длина дороги PQRS = 90 м

      Длина дороги ABCD = 60 м

      Ширина каждой дороги 90 = 03 м

      Площадь дорог = ar (PQRS) + ar (ABCD) − ar (KLMN)

      = (90 × 3) + (60 × 3) − (3 × 3)

      = 270 + 180 − 9 = 441 м 2

      Стоимость строительства 1 м 2 дороги = 110 рупий

      Стоимость строительства 441 м 2 дороги = 110 × 441 = 48510 рупий
       ​​​​​​

      Видео Решение для периметра и площади (Страница: 227 , Q.No.: 7)

      Решение NCERT для математики класса 7 — периметр и площадь 227 , Вопрос 7

      Стр. № 227:
      Вопрос 8:

      Прагья оберните шнур вокруг круглой трубы радиусом 4 см (примыкающей рисунок) и отрежьте необходимую длину шнура. Затем она завернула вокруг квадратной коробки со стороной 4 см (также показано). Были ли у нее какие-либо шнур остался? (π = 3,14).

      Ответ:

      Периметр стороны квадрата = 4 × Сторона квадрата = 4 × 4 = 16 см

      Периметр круглой трубы = 2π r = 2 × 3,14 × 4 = 25,12 см

      Длина хорда влево с Pragya = 25,12 − 16 = 9,12 см

      Страница № 227:
      Вопрос 9:

      соседний рисунок представляет собой прямоугольную лужайку с круглым цветком кровать посередине. Найти:

      (i) площадь всей земли

      (ii) площадь цветника

      (iii) площадь газона без учета площади клумбы

      (iv) окружность клумбы.

      Ответ:

      (i) Площадь всей земли = Длина × Ширина = 10 × 5 = 50 м 2

      (ii) Площадь клумбы = π r 2 = 3,14 × 2 × 2 = 12,56 м 2

      (iii) Площадь газона, исключая клумбу = площадь всей земли – площадь

      клумба

      = 50 − 12,56 = 37,44 м 2

      (iv) Окружность клумбы = 2π р = 2 × 3,14 × 2 = 12,56 м

      Страница № 227:
      Вопрос 10:

      На следующих рисунках найдите площадь заштрихованных частей:

      Ответ:

      EF05 из 900 = ar (ABCD) — ar (BCE) — ar (AFE)

      = (18 × 10) − (10 × 8) − (6 × 10)

      = 180 − 40 − 30 = 110 см 2
      ​​​​​​​​

      (ii)

      ar (QTU) = ar (PQRS) − ar (TSU) − ar (RUQ) − ar (PQT)

      = (20 × 20) −(10 × 10) − (20 × 10) −(20 × 10)

      = 400 − 50 − 100 − 100 = 150 см 2

      Видео Решение для периметра и площади (Страница: 227 , Q.

Добавить комментарий

Ваш адрес email не будет опубликован. Обязательные поля помечены *